Categories
Exam Questions Harvard

Harvard. Economics semester final examinations, 1900-01.

In the first full academic year of the twentieth century the Harvard economics department offered the following courses. The course links take you to the official course announcement, instructor names, enrollment figures, and the transcribed semester examinations.

Economics 1. Outlines of Economics
Economics 2. Economic Theory of the 19th Century
Economics 3. Principles of Sociology
Economics 5. Railways and Other Public Works
Economics 6. Economic History of the U.S.
Economics 8. Money
Economics 9. Labor Question in Europe and the U.S.
Economics 10. European Mediaeval Economic History
Economics 12. Banking and Leading Banking Systems
Economics 12a. International Payments and Gold/Silver Flows
Economics 13. Methods of Economic Investigation
Economics 17. Economic Organization and Resources in Europe
Economics 18. Principles of Accounting
Economics 19. General View of Insurance
Economics 20d. Adam Smith and Ricardo

__________________________________

Economics 1.
Outlines of Economics

Primarily for Undergraduates.

Course Announcement
  1. Outlines of Economics. , Wed., Fri., at 9. Professor Taussig, Dr. Sprague, Mr. Andrew, and Messrs. — and — .

Source: Harvard University. Announcement of the Courses of Instruction provided by the Faculty of Arts and Sciences for the Academic Year 1900-1901, p. 41.

Course Enrollment
  1. Professor [Frank W.] Taussig, Drs. [Oliver Mitchell Wentworth] Sprague and [Abram Piatt] Andrew, and Messrs. [Charles] Beardsley and [James Horace] Patten. — Outlines of Economics.

Total 442: 23 Seniors, 70 Juniors, 257 Sophomores, 29 Freshmen, 63 Others.

Source: Harvard University. Report of the President of Harvard College 1900-1901, p. 64.

1900-01
ECONOMICS 1
[Mid-year Examination]

Arrange your answers strictly in the order of the questions.

  1. In what manner do you think that (a) the individual efficiency of laborers, (b) their collective efficiency, would be affected by the general adoption of profit sharing? of socialism?
  2. It has been said that the original formation of capital is due to abstinence or saving, but its permanent maintenance is not. What do you say to either statement?
  3. Wherein is Walker’s presentation of the forces that make the general rate of wages better than Mill’s, wherein not so good?
  4. “The extra gains which any producer or dealer obtains through superior talents for business, or superior business arrangements, are very much of a similar kind. …All advantages, in fact, which one competitor has over another, whether natural or acquired, whether personal or the result of social arrangements, bring the commodity, so far, into the Third Class, and assimilate the advantage to a receiver of rent.”
    Explain (a) what is this Third Class, and what is the law of value applicable to it; (b) what Mill would say as to the proposition here stated; (c) what Walker would say?
  5. What qualifications of the general principle of rent can you state, in its application to (a) premises used for building purposes, (b) dwelling-houses, (c) mines?
  6. If all men had the same start in life, would there be differences of wages? If so, of what sort? If not, why not?
  7. “Since cost of production here fails us, we must revert to a law of value anterior to cost of production and more fundamental…” In what cases does cost of production fail us? Will “cost of reproduction” cover such cases? Is there another law more fundamental?
  8. Under what circumstances. if ever, will a general rise in wages affect the relative values of commodities? Would your answer be the same as to a general rise in profits?
  9. In what manner do you believe business profits, interest, and wages would be affected by the general adoption of the various forms of consumers’ coöperation? of producers’ cooperation?

Source: Harvard University Archives. Harvard University Mid-year Examinations, 1852-1943. Box 4, Bound volume: Examination Papers, Mid-Years, 1900-01.

1900-01
ECONOMICS 1
[Year-end Examination]

I.
Answer three.

  1. How will the value of land be affected by
    1. an increase in population,
    2. a reduction in the rate of interest,
    3. a protective tariff on agricultural produce.
  2. How will the price of grain be affected by
    1. a tax proportioned to the economic rent of the land,
    2. an equal tax upon all land.
  3. “Profits do not form a part of the price of the products of industry, and do not cause any diminution of the wages of labor.”
    Would Mill agree to this statement? Would you?
  4. Upon what does the general level of wages depend (a) according to Mill, (b) according to Walker? What would you expect these writers to say as to the effect of a protective tariff on the general level of wages?

II.
Answer two.

  1. If a country exports on a large scale a commodity not previously exported, will its other exports be affected? If so, how? If not, why not?
  2. Can a country have a permanently “unfavorable” balance of trade? If so, under what conditions? If not, why not?
    Can a country permanently export specie? If so, under what conditions? If not, why not?
    Can the rate of foreign exchange in a country be permanently at the specie-shipping point? If so, under what conditions? If not, why not?
  3. How would you expect the issue of a paper currency to effect foreign trade,—
    1. While the notes were still redeemable;
    2. After they had become irredeemable.

III.
Answer two.

  1. Define the following terms

Seignorage,
Clearing house loan certificates,
Silver Certificates,
United States notes,
Inconvertible paper.

  1. How would the adoption of bimetallism affect the stability of the value of money?
    1. according to Mill,
    2. according to Walker,
    3. in your own opinion.
  2. How is the value of money in a country likely to be affected by an increase in
    1. the quantity of commodities produced and sold,
    2. the quantity of bank notes,
    3. the volume of bank deposits.

Which of these changes would you expect to exercise most influence? Which least? Give your reasons.

IV.
Answer all.

  1. Compare and explain the operations of the Bank of England and those of the New York banks in a time of crisis,
  2. Arrange these items…

Government Securities 40.
Surplus 3.
Notes 38.
Specie 40.
Deposits 55.
Capital 14.
Loans 30.

    1. … in their proper order, as they would stand in an account of the Bank of France.
    2. … as they would stand in an account of a national bank of the United States; and state (1) whether this could be an account of a national bank, and (2) whether the proportions of the different items are such as you would be likely to find in an account of such a bank.
    3. … as they would stand in an account of the Bank of England, assuming the uncovered issue to be 17.

Source: Harvard University Archives. Harvard University, Examination Papers, 1873-1915. Box 5, Bound volume: Examination Papers, 1900-01. Papers Set for Final Examinations in History, Government, Economics, Philosophy, Education, Fine Arts, Architecture, Landscape Design, Music in Harvard College (June, 1901), pp. 21-23.

 

__________________________________

Economics 2.
Economic Theory
in the 19th Century

For Undergraduates and Graduates.

Course outline and readings.

Course Announcement
  1. Economic Theory in the Nineteenth Century. , Wed., Fri., at 2.30 Professor Taussig. [note: Professor Carver taught the course]

Source: Harvard University. Announcement of the Courses of Instruction provided by the Faculty of Arts and Sciences for the Academic Year 1900-1901, p. 41.

Enrollment
  1. Professor Carver. — Economic Theory in the Nineteenth Century.

Total 45: 6 Graduates, 15 Seniors, 16 Juniors, 5 Sophomores, 3 Other.

Source: Harvard University. Annual Reports of the President and Treasurer of Harvard College, 1900-01, p. 64.

1900-01
ECONOMICS 2
[Mid-year examination]
  1. Define value and explain why one commodity possesses more value in proportion to its bulk than another.
  2. Explain the various uses of the term diminishing returns, and define it as you think it ought to be defined.
  3. In what sense does a law of diminishing returns apply to all the factors of production.
  4. State briefly Böhm-Bawerk’s explanation of the source of interest.
  5. What, if any, is the relation of abstinence to interest.
  6. Would you make any distinction between the source of wages and the factors which determine rates of wages? If so, what? If not, why not?
  7. Discuss the question: Is a demand for commodities a demand for labor?
  8. What is the relation of the standard of living to wages.
  9. Discuss briefly the following questions relating to speculators’ profits. (a) Do speculators as a classmake any profits? (b) Are speculators’ profits in any sense earned?
  10. In what sense, if any, does the value of money come under the law of marginal utility?

Source: Harvard University Archives. Harvard University Mid-year Examinations, 1852-1943. Box 4, Bound volume: Examination Papers, Mid-Years, 1900-01.

1900-01
ECONOMICS 2
[Year-end Examination]

Discuss the following topics.

  1. The bearing of the marginal utility theory of value upon the questions of wages and interest.
  2. The definitions of capital as given by Taussig and Clark.
  3. Clark’s explanation of the place of distribution within the natural divisions of economics.
  4. Clark’s method of distinguishing between the product of labor and the product of capital.
  5. Clark’s distinction between rent and interest.
  6. Böhm-Bawerk’s theory of the nature of capital.
  7. The origin of capital, according to Böhm-Bawerk and Clark.
  8. The meaning of the word “productive” in the following proposition: “Protection is an attempt to attract labor and capital from the naturally more productive, to the naturally less productive industries.”
  9. The incidence of tariff duties.
  10. The theory of production and the theory of valuation as the two principal departments of economics.

Source: Harvard University Archives. Harvard University, Examination Papers, 1873-1915. Box 5, Bound volume: Examination Papers, 1900-01. Papers Set for Final Examinations in History, Government, Economics, Philosophy, Education, Fine Arts, Architecture, Landscape Design, Music in Harvard College (June, 1901), pp. 23-24.

 

__________________________________

Economics 3.
Principles of Sociology

For Undergraduates and Graduates.

Course Announcement
  1. The Principles of Sociology. — Development of the Modern State, and of its Social Functions. , Wed., and (at the pleasure of the instructor) Fri., at 1.30. Mr. —.

Source: Harvard University. Announcement of the Courses of Instruction provided by the Faculty of Arts and Sciences for the Academic Year 1900-1901, p. 41.

Enrollment
  1. Asst. Professor [Thomas Nixon] Carver. — The Principles of Sociology. Development of the Modern state, and of its Social Functions.

Total 57: 9 Graduates, 22 Seniors, 8 Juniors, 14 Sophomores, 4 Others.

Source: Harvard University. Annual Reports of the President and Treasurer of Harvard College, 1900-01, p. 64.

1900-01
ECONOMICS 3.
[Mid-year Examination]

Answer only ten questions.

  1. Upon what does Kidd base his argument that religion is necessary to keep men from taking such political action as would suspend economic competition, and what is the crucial point in his argument?
  2. In the light of Kidd’s theory of social evolution, discuss the question: Can there be a permanent civilization? Or, do the conditions which promote progress also ensure decay?
  3. Classify the sanctions for conduct which originate outside the individual and explain your classification.
  4. Explain and illustrate the meaning of the following: “Generalizing this struggle and extending it to every form existing in the social life — linguistic, religious, political, artistic, and moral, as well as industrial — we see that the really fundamental social opposition must be sought for in the bosom of the social individual himself.” (Tarde, Social Laws. Ch. II. p. 83.)
  5. What is meant by social stratification? How does it originate? What are some of its consequences?
  6. Compare Herbert Spencer’s theory of progress with Lester F. Ward’s, giving special attention to the argument which each offers in support of his theory.
  7. What, according to Patten, are the chief obstacles to a progressive evolution.
  8. Explain the following. “The difference between that society of conscious units which we call mind, and a society of human beings on our planet, is in the completeness of the mechanism.” (Patten, Theory of Social Forces. Ch. II. p. 21.)
  9. What, according to Patten, is the significance of the transition from a pain to a pleasure economy.
  10. How does Bagehot account for the origin of national traits?
  11. Discuss the question: Does charity retard the process of race improvement?

Source: Harvard University Archives. Harvard University Mid-year Examinations, 1852-1943. Box 4, Bound volume: Examination Papers, Mid-Years, 1900-01.

1900-01
ECONOMICS 3.
[Year-end Examination]

Discuss the following topics

  1. The definition of progress.
  2. Charity as a factor in human selection.
  3. The way in which, according to Spencer, the different classes of institutions are related to one another.
  4. The sanctions for conduct.
  5. A moral ideal as a factor in human selection.
  6. The natural antagonism of human interests and the problem of evil.
  7. The storing of the surplus energy of society.
  8. The influence of property on the relations of the sexes.
  9. Labor and service as bases of distributive justice
  10. The influence of militarism upon race development, or deterioration.

Source: Harvard University Archives. Harvard University, Examination Papers, 1873-1915. Box 5, Bound volume: Examination Papers, 1900-01. Papers Set for Final Examinations in History, Government, Economics, Philosophy, Education, Fine Arts, Architecture, Landscape Design, Music in Harvard College (June, 1901), p. 24.

 

__________________________________

Economics 5.
Railways and Other Public Works

For Undergraduates and Graduates.

Course Announcements

51 hf. Railways and Other Public Works, under Public and Corporate Management. Half-course (first half-year). Tu., Th. and (at the pleasure of the instructor) Sat., at 1.30. Mr. Meyer.

52 hf. Railways and Other Public Works (advanced course). Half-course (second half-year). Tu., Th. and (at the pleasure of the instructor) Sat., at 1.30. Mr. Meyer.

Source: Harvard University. Announcement of the Courses of Instruction provided by the Faculty of Arts and Sciences for the Academic Year 1900-1901, p. 42.

Enrollments

[Economics] 51 hf. Mr. [Hugo Richard] Meyer.— Railways and other Public Works, under Public and Corporate Management.

Total 86: 4 Graduates, 52 Seniors, 17 Juniors, 4 Sophomores, 9 Others.

[Economics] 52 hf. Mr. Meyer.— Railways and other Public Works (advanced course).

Total 9: 3 Graduates, 4 Seniors, 1 Junior, 1 Sophomore.

Source: Harvard University. Annual Report of the President of Harvard College, 1900-1901, p.64.

1900-01
ECONOMICS 51
[Mid-year Examination]

Omit the last question if the paper seems too long

  1. The construction put upon the long and short haul clause: by the Interstate Commerce Commission; by the Supreme Court.
  2. The decisions of the Interstate Commerce Commission on group rates.
  3. The railway rate situation in Germany [Prussia]; does it throw any light on the railway problem in the United States?
  4. “If pooling produces any beneficial result, it necessarily does so at the expense of competition. It is only by destroying competition that the inducement to deviate from the published rate is wholly removed….By the legalizing of pooling the public loses the only protection which it now has against the unreasonable exactions of transportation agencies.”—Give your reasons for accepting or rejecting this statement.
    Alternative:—
    The reasons for the instability of pools in the United States.
  5. The Iowa Railroad Commission.
    Alternative:—
    To what extent was the long and short haul clause of the Interstate Commerce Act enforced; what was the effect of that enforcement: on railway revenues; on intermediate shipping or distributing points?
  6. The body of administrative law to be found in the decisions of the Massachusetts Gas and Electric Light Commission’s decisions upon petitions for reductions in the price of gas.
  7. (a) Is it to the public interest to insert in street railway charters provisions seeking to secure to the municipality or the state a share in any excess of profit over the normal rate?
    Alternative: (b) and (c).
    (b) The evidence as to the return on capital obtainable in street railway ventures.
    (c) What questions of public policy were raised in the case of the Milwaukee Street Railway and Electric Light Co. vs. The City of Milwaukee?
  8. What statistics were used in illustrating in a general way the statement that railway charges are based upon what the traffic will bear; in discussing the bearing of stock-watering upon railway rates; in discussing the return obtained by capital invested in railway enterprises in the United States?

Source: Harvard University Archives. Examination Papers, 1873-1915. Box 5. Bound Volume: Examination Papers 1900-01. Papers Set for Final Examinations in History, Government, Economics, Philosophy, Education, Fine Arts, Architecture, Landscape Design, Music in Harvard CollegeJune, Pages 24-25.

1900-01
ECONOMICS 52
[Year-end Examination]
  1. The railways and the national finances in Prussia and Australia.
  2. Railway rates and the export trade of the United States since 1893, or, 1896.
  3. The economic situation in Australia since 1892, and the Australian railways.
  4. “A fatal objection to the income or preference bond is that it is an attempt to combine two contradictory commercial principles.”
    Discuss this statement fully. What does it mean? Is it true?
  5. If you had access to all the accounts of a railroad, how should you determine the value to it of one of its branch lines?
  6. To what accounts would you charge the following expenditures? (If you do not remember the exact Interstate Commerce Commission classification, use your best judgment.) State reasons in each case.
    Engineer’s wages on a special train conveying the general manager to an extensive flood covering the line.
    Fireman’s wages on an engine employed exclusively in switching to and from the repair shops.
    Conductor’s wages on a worktrain engaged in taking up rails on an abandoned branch.
    Brakeman’s wages on a train engaged solely in hauling company’s coal for company’s use.
    Cost of taking up comparatively new sound rails judged too light for heavy rolling stock.
    Cost at a competitive point of a new station to replace an old one which was large enough but old-fashioned.
  7. State the commonest problems facing a reorganization committee for an insolvent road, and then suggest and defend one course of procedure for each problem.
  8. Combine and arrange the following items so as to give the best information about the operation and condition of the road. (Do not rewrite the names but use the corresponding numbers where possible.)
1. Passenger train miles 2,000,000
2. Freight train miles 3,400,000
3. Passenger train earnings $2,400,000
4. Freight train earnings $5,500,000
5. Income from investments $100,000
6. Dividends $500,000
7. Operating expenses $4,700,000
8. Av. no. pass. cars per train 4
9. Av. no. passengers per car 11
10. Tons freight carried 2,800,000
11. Av. load per car (loaded and empty), tons 8.2
12. Av. no. loaded cars per train 12.3
13. Av. no. empty cars per train 6.7
14. Interest charge for year $2,200,000
15. Due other roads $100,000
16. Stocks and bonds owned $4,900,000
17. Supplies on hand $500,000
18. Taxes for the year $300,000
19. Accounts receivable $500,000
20. Cash $1,000,000
21. Surplus for the year $300,000
22. Profit and loss account $1,000,000
23. Taxes accrued but not due $100,000
24. Capital stock $50,000,000
25. Interest due $700,000
26. Funded debt $45,000,000
27. Due from other roads $100,000
28. Interest accrued not due $300,000
29. Franchises and property $90,400,000
30. Bonds of the company in its treasury $800,000
31. Accounts payable $1,000,000
32. No. of passengers carried 2,300,000

Source: Harvard University Archives. Examination Papers, 1873-1915. Box 5. Bound Volume: Examination Papers 1900-01. Papers Set for Final Examinations in History, Government, Economics, Philosophy, Education, Fine Arts, Architecture, Landscape Design, Music in Harvard College. June, 1901. Pages 25-27.

 

__________________________________

Economics 6.
Economic History of the U.S.

For Undergraduates and Graduates.

Course Announcement
  1. The Economic History of the United States. Tu., Th., at 2.30, and a third hour at the pleasure of the instructor. Mr. —.

Source: Harvard University. Announcement of the Courses of Instruction provided by the Faculty of Arts and Sciences for the Academic Year 1900-1901, p. 42.

Enrollment
  1. Professor Taussig. — The Economic History of the United States.

Total 164: 9 Graduates, 63 Seniors, 68 Juniors, 13 Sophomores, 11 Others.

Source: Harvard University. Annual Reports of the President and Treasurer of Harvard College, 1900-01, p. 64.

1900-01
ECONOMICS 6
[Mid-year Examination]

Arrange your answers strictly in the order of the questions. Answer all the questions,

  1. The nature and object of the scales of depreciation established by Congress and by the States at the close of the war of the Revolution; and how far these objects were accomplished.
  2. “The year 1789 marks no such epoch in economies as it does in political history.” — Taussig. How far is this true as to (1) financial legislation; (2) tariff legislation; (3) the course of foreign trade; (4) the growth of manufactures?
  3. Explain how you would distinguish Treasury Notes designed to circulate as currency from those designed simply to meet financial needs; and state when and under what circumstances, between 1789 and 1860, the United States resorted to issues of the first kind.
  4. Suppose the charter of the first Bank of the United States had been renewed: would the effect have been favorable or unfavorable for the finances of the government, for the bank, for the community, in 1812-1815?
  5. Suppose the charter of the second Bank of the United States had been renewed: would the effect have been favorable or unfavorable for the finances of the government, for the bank, for the community, in 1835-40?
  6. Describe the Independent Treasury system, as first established and as finally settled (give dates). Do you believe it better than the alternative system proposed by its opponents? Why?
  7. The causes of the crises of 1837 and 1857: wherein similar, wherein different.
  8. State what were the duties on cotton goods in 1809, 1819, 1839, 1859; and give your opinion whether the duties at these several dates were designed to give protection, and whether protection was then expedient.
  9. Why the early railway enterprises of the States were undertaken as public enterprises; and how far their history may be fairly cited for or against the policy of public management.

Source: Harvard University Archives. Harvard University Mid-year Examinations, 1852-1943. Box 4, Bound volume: Examination Papers, Mid-Years, 1900-01.

1900-01
ECONOMICS 6
[Year-end Examination]

Arrange your answers strictly in the order of the questions

  1. Explain summarily at what dates and to what extent land-grants and bond-subsidies were extended to railways by the United States; and state whether you believe these measures brought advantage to the country.
  2. Was the management of the finances during the Civil War fraught with more or less evil consequences than that during the War of 1812, as regards (1) the currency, (2) the banks?
  3. State what main sources of revenue were expected to be used, what were used in fact, by the United States in each of the years 1862, 1863, 1864; and explain how the resort to the sources actually used came about.
  4. For the decade 1870-80, explain the connection between the course of prices, foreign trade, railway operations, and currency legislation.
  5. For the decade 1880-90, connect the history of the public debt, the national revenues, the banking system, the silver currency.
  6. Does the argument for protection to young industries find support in the history of (1) the cotton manufacture before 1830, (2) the silk manufacture since 1870, (3) the tin plate industry since 1890.
  7. Explain how the theory of comparative costs may be applicable to the present situation as regards carpet wool, beet sugar, glassware, woollen cloths (take three).
  8. What changes were made in the duties on raw and refined sugar in 1890, 1894, 1897? Which mode of treatment do you regard the most advisable, and why?
  9. State what causes you believe to have chiefly promoted the growth and maintenance of the sugar and oil combinations; and consider which of these two you regard as typical, and as instructive for forecasting the future of combinations.

Source: Harvard University Archives. Examination Papers, 1873-1915. Box 5. Bound Volume: Examination Papers 1900-01. Papers Set for Final Examinations in History, Government, Economics, Philosophy, Education, Fine Arts, Architecture, Landscape Design, Music in Harvard College. June, 1901. Pages 27-28.

 

__________________________________

Economics 81
Money

For Undergraduates and Graduates.

Course Announcement

81 hf. Money. Half-course (first half-year). Tu., Th., Sat., at 11. Mr. Andrew.

Source: Harvard University. Announcement of the Courses of Instruction provided by the Faculty of Arts and Sciences for the Academic Year 1900-1901, p. 42.

Enrollment

81 hf. Dr. [Abram Piatt] Andrew. Money.

Total 122: 3 Graduates, 56 Seniors, 41 Juniors, 8 Sophomores, 1 Freshman, 13 Others.

Source: Harvard University. Annual Reports of the President and Treasurer of Harvard College, 1900-01, p. 64.

1900-01
ECONOMICS 8
[Mid-year Examination]

Answer only three questions from each group, but consider the questions strictly in the order of their arrangement 

I

  1. What is meant by

(1) a “double” standard;
(2) a “parallel” standard;
(3) a “limping” standard;
(4) a “single” standard;

Cite at least two historic examples of each, giving approximate dates.

  1. The following are estimates which have been made of the average production of silver, and its annual average export to the Orient in millions of ounces:

Production Export to East
1851-55 28 mill.

20 mill.

1855-60

29  ” 52  ”
1861-65 35  ”

53  ”

1865-70

43  ”

25  ”

State the causes of the singular situation revealed in these figures, and explain its actual effect upon the relative values of gold and silver in Europe.

  1. Suppose that the British government in 1870 had used the right conferred by the act of 1816, and had proclaimed the free coinage of silver at the ratio then current. What differences do you think would have occurred in the subsequent currency history of the world?
  2. Describe the effect of the suspension of the coinage of silver upon the value of the currency in each of the following cases:—
    (1) in Holland; (2) in Austria; (3) in Russia; (4) in India.

II

  1. “Before 1873 we had coined in the United States only about eight million silver dollars ($8,031,238) while since the date fixed as the beginning of demonetization we have coined nearly five hundred millions ($485,427,703).”
    How do you explain (1) the small amount of dollars coined before 1873? (2) the large amount coined since then?
  2. What in your opinion was the real significance of (1) the act of 1803? (2) the act of 1873?
  3. “With the exception of the brief period of fifteen years (1544-60) the English coins have never been debased.”
    In what sense and to what extent is this statement correct?
  4. In writing of the currency history of England during the years, immediately succeeding the great recoinage (1696) Mr. Dana Horton says:—
    “And so the full weight standard coin of the Realm, to create a stock of which the State had spent a sum greater than its regular annual revenue, and equal to perhaps a fourth of the country’s total stock of cash, — was allowed to find its way back to the melting-pot in exchange for cheaper gold.”
    Explain the situation to which he refers, and the reasons for this disappearance of the “standard coin.”

III

  1. (a) What were the main arguments which Lord Liverpool advanced in favor of a single gold standard?
    (b) What were the legislative acts in which his influence is to be traced?
  2. (a) Do falling prices necessarily mean an increase in the burden of debts?
    (b) Do they in the long run inevitably diminish the productiveness of industry?
    (c) To what extent are they prejudicial to the interests of the working classes?
  3. “It is generally agreed that every fall in the value of silver acted at the time as a stimulus to Indian exports and as a check on imports into India.”
    (1) Explain this statement, (2) state how far it is confirmed by commercial statistics, and (3) show whether such a condition is ever likely to be of prolonged duration.
  4. It is alleged that the Russian government, by stimulating exports, and hindering imports, has endeavored to secure a favorable balance of trade, with the idea of increasing the quantity of gold in the country? What do you think would be the ultimate effect of such a policy if continuously pursued?

Source: Harvard University Archives. Harvard University Mid-year Examinations, 1852-1943. Box 4, Bound volume: Examination Papers, Mid-Years, 1900-01.

Also: Harvard University Archives. Examination Papers, 1873-1915. Box 5. Bound Volume: Examination Papers 1900-01. Papers Set for Final Examinations in History, Government, Economics, Philosophy, Education, Fine Arts, Architecture, Landscape Design, Music in Harvard College (June, 1901), pp. 28-30.

 

__________________________________

Economics 9.
The Labor Question in Europe and the U.S.

For Undergraduates and Graduates.

Taught by W. F. Willoughby (Edward Cummings’ successor).

Course Announcement
  1. The Labor Question in Europe and the United States. — The Social and Economic Condition of Workingmen. Tu., Th., Sat., at 10. Mr. —.

Source: Harvard University. Announcement of the Courses of Instruction provided by the Faculty of Arts and Sciences for the Academic Year 1900-1901, p. 42.

Enrollment

92 hf. Mr. W. F. Willoughby. — The Labor Question in Europe and the United States. The Social and Economic Condition of Workingmen.

Total 146: 3 Graduates, 53 Seniors, 40 Juniors, 35 Sophomores, 3 Freshmen, 12 Others.

Source: Harvard University. Annual Reports of the President and Treasurer of Harvard College, 1900-01, p. 64.

1900-01
ECONOMICS 9
[Year-end examination]
  1. Show how the change in the organization of industry from the handicraft system and production on a small scale to the factory system and production on a large scale has led to; (a) efforts to supplant the wages system by socialism[,] coöperation, etc., (b) the trade union movement, and (c) compulsory compensation acts.
  2. Give the arguments for and against profit-sharing as regards (a) it being a more just system of enumeration than the wages system, and (b) its practical advantages.
  3. What are the two systems of coöperative production now practice in Great Britain, and why are they meeting with more success than earlier efforts?
  4. Describe the trade agreement between the National Metal Trades Association and the International Association of Machinists in such a way as to show its essential character and significance, and particularly its relation to the trade union movement and the question of the prevention and adjustment of industrial disputes.
  5. What was the nature of the “new unionism” movement in Great Britain, and its success?
  6. What is the general character of the Massachusetts State Board of Conciliation and Arbitration; what its duties and its powers?
  7. Describe the essential features of the French Workmen’s Compensation Act.
  8. Give a brief sketch of the Social Democratic Party in Germany, with the names of its early leaders and important events in its history.
  9. In what ways can the municipality take action for the improvement of the housing condition of the poorer classes without itself building tenements? What are some of the objections to the municipalities themselves undertaking building operations?
  10. Show why employment bureaus can do but little for the solution of the general problem of the unemployed.

Source: Harvard University Archives. Examination Papers, 1873-1915. Box 5. Bound Volume: Examination Papers 1900-01. Papers Set for Final Examinations in History, Government, Economics, Philosophy, Education, Fine Arts, Architecture, Landscape Design, Music in Harvard College (June, 1901), pp. 30-31.

Enrollment (Economics 9a)

9a2 hf. Mr. W. F. Willoughby. — Provident Institutions. Workingmen’s Insurance, Friendly Societies, Savings Banks.

Total 22: 1 Graduate, 13 Seniors, 5 Juniors, 2 Sophomores, 1 Other.

Source: Harvard University. Annual Reports of the President and Treasurer of Harvard College, 1900-01, p. 64.

1900-1901
ECONOMICS 9a
[Year-end Examination]
  1. What is the general situation in France at the present time in respect to insurance against old age and invalidity? Describe briefly the organization and workings of important institutions, and show particularly how the government is attempting to further this kind of insurance.
  2. What has been the general policy of the British government in respect to the regulation of Friendly Societies? Give the main features of law now regulating them.
  3. Describe the Fraternal Beneficial Orders of the United States as regards (a) their general scheme of organization, and (b) system of insurance.
  4. Show wherein this insurance system is defective by contrasting it with that of ordinary life insurance companies: indicate reforms that are necessary and how they can best be brought about.
  5. Contrast the systems of savings banks in Great Britain and the United States.
  6. In what respects are coöperative banks of the Schulze-Delitzsch and Raiffeisen type more valuable social institutions than the ordinary savings banks?
  7. Describe the principles upon which all coöperative building and loan associations in this country are organized, and indicate ways in which they might profitably be subjected to more rigid state control.
  8. Why is it impracticable to insure against unemployment?
  9. Outline briefly the system of sick insurance in Germany.

Source: Harvard University Archives. Examination Papers, 1873-1915. Box 5. Bound Volume: Examination Papers 1900-01. Papers Set for Final Examinations in History, Government, Economics, Philosophy, Education, Fine Arts, Architecture, Landscape Design, Music in Harvard College (June, 1901), p. 31.

 

__________________________________

Economics 10.
Mediaeval Economic History of Europe.

For Undergraduates and Graduates.

Course Announcement

For Undergraduates and Graduates.

  1. The Mediaeval Economic History of Europe. Tu., Th., and (at the pleasure of the instructor) Sat., at 12. Professor Ashley.

Source: Harvard University. Announcement of the Courses of Instruction provided by the Faculty of Arts and Sciences for the Academic Year 1900-1901, p. 41.

Enrollment
  1. Professor Ashley. The Mediaeval Economic History of Europe.

Total 11: 6 Graduates, 4 Seniors, 1 Junior.

Source: Harvard University. Annual Reports of the President and Treasurer of Harvard College, 1900-01, p. 64.

1900-01
ECONOMICS 10
[Mid-year Examination]

Not more than six questions should be attempted, of which the first should be one.

  1. Translate, and briefly comment upon
    1. Toto regis Willelmi primi tempore perseveravit haec institutio, usque tempora regis Henrici filii ejus; adeo ut viderim ego ipse quosdam qui victualia statutis temporibus de fundis regiis ad curiam deferri viderint.
    2. In Kateringes sunt X hidae ad geldum Regis. Et de istis X hidis tenent XL villani XL virgas terrae.
    3. Compotus Roberti Oldeman praepositi de Cuxham, ab in crastino Sancti Jacobi anno regni Regis Edwardi filii Regis Edwardi decimo usque ad in crastinum Sancti Jacobi proxime sequentis anno regni Regis Edwardi praedicti undecimo intrante.
    4. Rogamus . . . ademptum sit jus etiam procuratoribus nedum conductori adversus colons ampliandi partes agrarias aut operarum praebitionem jugorumve.
    5. Orgeterix ad judicium omnem suam familiam, ad hominum milia decem, undeque coëgit et omnes clientes obaeratoque suos quorum magnum numerum habebat eodem conduxit.
  2. What materials have we for forming a judgment as to the position of the rural population of England in the period from the eleventh to the fourteenth centuries? Classify them, and indicate the value of each class for the purposes of this enquiry.
  3. Wherein did the status of the coloni of the later Roman Empire resemble or differ from that of the medieval villein?
  4. Describe the constitution and working of manorial courts. What light does their history throw on the evolution of social classes?
  5. “Wie das Wort Dorf … dem Sinne nach einen Haufen bezeichnet, so ist auch haufenförmig oder Haufendorf der geeignetste Ausdruck für diese Art der Dorfenlage.” Explain and comment.
  6. “M. Fustel took for his point of departure the Provincial villa; Dr. Hildebrand takes the Kirghises of modern Asia.” Explain, and then show the peculiar dangers of each method.
  7. “We may safely follow Palgrave in taking the Anglo-Saxon townships as the integral molecules out of which the Anglo-Saxon State was formed.” Why? or why not?
  8. What was the gwely? What bearing has it on the general problem of “tribal” organization?
  9. What are the assumptions or postulates of modern Political Economy? To what extent were they true of the Middle Ages?
  10. Which book read in connection with this course has interested you most? Describe its method and estimate the value of its contribution to economic history.

Source: Harvard University Archives. Harvard University Mid-year Examinations, 1852-1943. Box 4, Bound volume: Examination Papers, Mid-Years, 1900-01.

1900-01
ECONOMICS 10
[Year-end Examination]

Not more than six questions should be attempted, of which the first should be one

  1. Briefly comment upon the following passages, and translate such of them as are not in English:—
    1. Colunt discreti ac diversi, ut fons, ut campus, ut nemus placuit. Vicos locant non in nostrum morem connexis et cohaerentibus aedificiis: suam quisque domum spatio circumdat.
    2. If a man agree for a yard of land or more, at a fixed rent, and plough it; if the lord desire to raise the land to him to service and to rent, he need not take it upon him, if the lord do not give him a dwelling.
    3. Ego S. … et ego P. … aliquantulam agri partem pro remedio animarum nostrorum W. episcopo in dominio donare decrevimus; id est xxx cassatorum in loco qui dicitur T.
    4. Si quis super alterum in villa migrare voluerit, et unus vel aliqui de ipsis qui in villa consistunt eum suscipere voluerit, si vel unus extiterit qui contradicat, migrandi ibidem licentiam non habebit.
    5. Qui habebant de tenentibus per diaetas totius anni, ut assolet de nativis, oportebat eos relaxare et remittere talia opera.
    6. If any one does an injury who is not of the gild and is of the franchise … he shall lose his franchise.
  2. Explain the position of Maitland’s Domesday Book and Beyond in the discussion concerning the origin of the manor.
  3. Distinguish between the several characteristics of mediaeval towns, and indicate the part played by each, in your opinion, in the formation of specifically urban conditions.
  4. Examine the relations between questions of personal status and questions of economic condition in relation to the ‘peasants’ of the Middle Ages.
  5. What is the nature of our evidence as to the Peasants’ Rising of 1381? Is there any reason for ascribing anything like an economic programme to the leaders of the movement?
  6. Indicate briefly (a), the several influences tending towards a corporate organization of industry in the later Middle Ages and (b) the advantages or disadvantages of such an organization.
  7. Distinguish between the several immigrations of foreign work people to England before the accession of James I, and explain the nature of their contributions to the development of English manufactures.
  8. The relation of John Major and Juan Vives to the development of the English ‘Poor Law.’
  9. What changes, if any, did the Reformation bring about in social life?

Source: Harvard University Archives. Examination Papers, 1873-1915. Box 5. Bound Volume: Examination Papers 1900-01. Papers Set for Final Examinations in History, Government, Economics, Philosophy, Education, Fine Arts, Architecture, Landscape Design, Music in Harvard College (June, 1901), pp. 32-33.

 

__________________________________

Economics 122.
Banking and the History of the Leading Banking Systems.

For Undergraduates and Graduates.

Course Announcement

122 hf. Banking and the history of the leading Banking Systems. Half-course (second half-year). Tu., Th., Sat., at 11. Dr. Sprague.

Source: Harvard University. Announcement of the Courses of Instruction provided by the Faculty of Arts and Sciences for the Academic Year 1900-1901, p. 43.

Enrollment

122 hf. Dr. [Oliver Mitchell Wentworth] Sprague. — Banking and the History of the Leading Banking Systems.

Total 128: 4 Graduates, 51 Seniors, 43 Juniors, 16 Sophomores, 14 Others.

Source: Harvard University. Report of the President of Harvard College, 1900-1901, p. 64.

1900-01
ECONOMICS 122
[Year-end Examination]

Arrange your answers strictly in the order of the questions. Answer all the questions under A and two of those under B

A

  1. Explain in detail and under different circumstances the effect of an advance of the rate of discount by the Bank of England upon the money market of London and upon the foreign exchanges.
  2. Taking the separate items of a bank account point out how those of the Bank of Amsterdam differed from those of a modern bank.
  3. Define and explain:—
    1. Bill broker.
    2. Banking Principle.
    3. The State Bank of Indiana.
    4. The banking law of Louisiana.
    5. Clearing House Certificates.
  4. The extent and banking consequences of government control of the Bank of France and the Reichsbank.
  5. How do government receipts and expenditures affect the money market (a) of London, (b) of New York?
  6. Explain with illustrations from the crises of 1857 and 1893 the nature of the demand for cash in time of crisis, and consider how far that demand may be met under a flexible system of note issue.

B

  1. (a) How far and with what qualifications may banking experience in the United States before 1860 be appealed to in the discussion of changes in our banking system? (b) How far, similarly, may Canadian experience be applied?
  2. “Why compel banks to send home for redemption a multitude of notes which can as well be used in payments and are sure to be reissued at once? Why impede the free use of its power of circulation by any enterprising bank by requiring the early redemption of notes which the holder does not in fact care or need to have redeemed?”
    Explain from past experience what regulations may be expected to bring about these results, and give the reasons for demanding them.
  3. Discuss the question of branch banking with reference to the United States, including in your discussion considerations of safety and economy. Would branch banking be more desirable than at present if notes were issued against general banking assets.

Source: Harvard University Archives. Examination Papers, 1873-1915. Box 5. Bound Volume: Examination Papers 1900-01. Papers Set for Final Examinations in History, Government, Economics, Philosophy, Education, Fine Arts, Architecture, Landscape Design, Music in Harvard College (June, 1901), pp. 34-35.

 

__________________________________

Economics 12a1.
International Payments and the Flow of Precious Metals.

For Undergraduates and Graduates.

Course Announcement

12a2 hf. International Payments and the Flow of the Precious Metals. Half-course (second half-year). Three times a week. Mr—.

Source: Harvard University. Announcement of the Courses of Instruction provided by the Faculty of Arts and Sciences for the Academic Year 1900-1901, p. 43.

Enrollment

[Economics] 12a1 . Mr. [Hugo Richard] Meyer.—International Payments and the Flow of the Precious Metals.

Total 16: 2 Graduates, 9 Seniors, 4 Juniors, 1 Other.

Source: Harvard University. Report of the President of Harvard College, 1900-1901, p. 64.

1900-01
ECONOMICS 12a1.
[Mid-Year Examination]

Observe strictly the order in which the questions are arranged.

  1. Sidgwick’s criticisms on Mill’s doctrine of international trade and their validity.
  2. What temporary changes in the general level of prices in this country should you expect to see, as the result of a large permanent withdrawal of foreign capital? What ultimate change of prices should you expect?
  3. Suppose the exportation of specie from the United States to be prohibited (or, as has sometimes been suggested, to be slightly hindered), what would be the effect on rates of exchange, and on prices of goods, either domestic or foreign? Would the country be a loser or not? [See Ricardo (McCulloch’s ed.), page 139.]
  4. The conditions which led to the flow of gold to the United States in the fiscal years 1880 and 1881?
  5. What economic conditions or events tended to make the year 1890 a turning point both in domestic and in international finance?

Alternative:

The reasons for the return flow from Europe of American securities in the years 1890-1900?

  1. What sort of wealth did France actually sacrifice in paying the indemnity? What was the process?
  2. Is Mr. Clare justified in making the general statement that “the gold-points mark the highest level to which an exchange may rise, and the lowest to which it may fall”?
  3. Why is it that certain trades bills are drawn chiefly, or even exclusively, in one direction, e.g. by New York on London and not vice versa; and how is this practice made to answer the purpose of settling payments which have to be made in one direction?

Alternative:

Why has England become the natural clearing-house for the world?

Source: Harvard University Archives. Harvard University Mid-year Examinations, 1852-1943. Box 4, Bound volume: Examination Papers, Mid-Years, 1900-01.

Also: Harvard University Archives. Examination Papers, 1873-1915. Box 5. Bound Volume: Examination Papers 1900-01. Papers Set for Final Examinations in History, Government, Economics, Philosophy, Education, Fine Arts, Architecture, Landscape Design, Music in Harvard College (June, 1901), pp. 33-34.

 

__________________________________

Economics 13.
Methods of Economic Investigation.

Primarily for Graduates.

Course Announcement
  1. Methods of Economic Investigation.—English Writers. German Writers. Tu., Th., at 1.30. Professor Taussig.
    Courses 15 and 13 are usually given in alternate years.

[15. The History and Literature of Economics to the close of the Eighteenth Century. Mon., Wed., and (at the pleasure of the instructor) Fri., at 12. Professor Ashley.
Omitted in 1900-01.]

Source: Harvard University. Announcement of the Courses of Instruction provided by the Faculty of Arts and Sciences for the Academic Year 1900-1901, p. 43.

Enrollment
1900-01

Economics 132 hf. Asst. Professor Carver. — Methods of Economic Investigation.

Total 10: 4 Graduates, 6 Seniors.

Source: Harvard University. Report of the President of Harvard College, 1900-1901, p. 64.

1900-01
ECONOMICS 13
[Year-end Examination]

Discuss ten of the following topics.

  1. The subdivision of economics into departments.
  2. The fields for the observation of economic phenomena.
  3. The place of historical and statistical research in economic investigation.

4, 5, 6. The methods of investigating:

    1. The causes of poverty.
    2. The effect of immigration on the total population of the United States.
    3. The effect of protection on the production of flax fibre, on the iron industry, or on any other industry which you may select.
  1. The nature of an economic law.
  2. The relation of the theory of probabilities to economic reasoning.
  3. The use of hypotheses in economic reasoning.
  4. The use of the terms “static” and dynamic in economic discussion.
  5. The use of diagrams and mathematical formulae in economic discussion.

Source: Harvard University Archives. Examination Papers, 1873-1915. Box 5. Bound Volume: Examination Papers 1900-01. Papers Set for Final Examinations in History, Government, Economics, Philosophy, Education, Fine Arts, Architecture, Landscape Design, Music in Harvard College (June, 1901), p. 35.

 

__________________________________

Economics 17.
Economic Organization and Resources of European Countries.

For Undergraduates and Graduates.

Course Announcement
  1. The Economic Organization and Resources of European countries. Mon., Wed., and (at the pleasure of the instructor) Fri., at 12. Professor Ashley.

Source: Harvard University. Announcement of the Courses of Instruction provided by the Faculty of Arts and Sciences for the Academic Year 1900-1901, p. 42.

Enrollment
  1. Professor Ashley. The Economic Organization and Resources of European countries.

Total 34: 5 Graduates, 14 Seniors, 9 Juniors, 3 Sophomores, 3 Others.

Source: Harvard University. Annual Reports of the President and Treasurer of Harvard College, 1900-01, p. 64.

1900-01
ECONOMICS 17
[Mid-year Examination]

Not more than eight questions should be attempted

  1. “It is less important for a particular community than ever it was to be in possession of cheap food and raw materials produced within its own domain.” Discuss this proposition.
  2. Describe very briefly the main features of the physical geography of England (illustrating your answer, if possible, with a map) and indicate in general terms their economic consequences.
  3. Set forth some of the general considerations which should be taken into account in answering the question whether the industrial development of Ireland has been injuriously affected by English legislation.
  4. Compare the number and character of the several classes maintained by agriculture in England, with those of the agricultural classes in the U.S. and on the continent of Europe.
  5. Explain the powers of dealing with his estate enjoyed at present by an English tenant for life under a settlement.
  6. What districts of England are now suffering most severely from agricultural depression, and why?
  7. Can any lessons be drawn for the U.S. from the recent history of productive coöperation in England? Give your reasons.
  8. Give a rapid survey of the apparent coal resources of the world.
  9. What points of especial interest are there to the economist in the history, situation, character, etc. of the South Wales Coal Field?
  10. What is meant by Collective Bargaining? What are its prerequisites? What examples of it are you acquainted with in America?

Source: Harvard University Archives. Harvard University Mid-year Examinations, 1852-1943. Box 4, Bound volume: Examination Papers, Mid-Years, 1900-01.

1900-01
ECONOMICS 17
[Year-end Examination]

Not more than eight questions should be attempted

  1. The British Chancellor of the Exchequer proposes to levy a duty of one shilling per ton upon the export of coal from the United Kingdom: He argues that the tax will not be borne by the producer, but mainly, if not wholly, by the foreign consumer. Consider (a) what are the conditions under which this is likely to be the case, (b) how far these conditions are at present realized in the case of England.
  2. Distinguish the successive stages in the technological history of iron and steel, and connect them with the industrial development of the several countries concerned.
  3. What were the questions at issue in England in the Engineering dispute of 1897? What, with your present knowledge, do you think ought to have been your attitude, had you then been (a) an English engineering employer, (b) a leading official of the employees’ union.
  4. Give a brief account of the organization of the English cotton manufacture (as distinguished from the securing either of the material or of a market for the product). Contrast it with American conditions; and consider how England and New England are likely to be affected by the growth of the manufacture in the Southern States.
  5. Distinguish between the several forms of capitalist combination at present to be observed in England. What general causes have led to the movement? What, if any, advantages does it promise, and what, if any, dangers does it threaten?
  6. Compare Bradford and Roubaix in any aspects which seem to you worthy of attention.
  7. “Lorsque il n’y a point d’hommes riches qui aient de gros capitaux à mettre dans les entreprises d’agriculture, lorsque les récoltes ne suffisent pas pour assurer aut entrepreneurs des profits égaux à ceux qu’ils tireraient de leur argent en l’employant de toute autre manière, on ne trouve point de fermiers qui veuilient louer les terres. Les propriétaires sont forcées de les faire cultiver par les métayers hors d’état de faire aucunes avances et de bien cultiver. Le propriétaire fait lui-même des avances médiocres qui lui produisent un très médiocre revenu.”
    Translate the passage from Turgot; and then consider how far his description applies to existing conditions in France and Italy.
  8. Show the relation of the great manufacturing industries of France to the distribution of coal in that country.
  9. Would the construction of the Rhine-Elbe canal be a benefit to Germany? Give your reasons.
  10. “Wir müssen uns Rechenschaft ablegen, ob ohne eine grössere Macht zur See, ohne eine solche die unsere Küsten vor Blockaden schützt, unseren Kolonialbesitz und unseren Welthandel absolut sicher stellt, unsere wirtschaftliche Zukunft gesichert sei.”
    Are there sufficient reasons in the contemporary situation of Germany for this anxiety on the part of Professor Schmoller?
  11. (a) Give a brief account of the contents, and then (b) compare the method and general attitude toward the subject, of von Schulze-Gaevernitz’s Social Peace and de Rousers’ Labour Question in Britain.
  12. What in the light of the experience in the English coal, iron, and cotton industries, would seem to you the most satisfactory form to be taken by joint wage agreements in the great industries of America?

Source: Harvard University Archives. Examination Papers, 1873-1915. Box 5. Bound Volume: Examination Papers 1900-01. Papers Set for Final Examinations in History, Government, Economics, Philosophy, Education, Fine Arts, Architecture, Landscape Design, Music in Harvard College (June, 1901), pp. 36-37.

 

__________________________________

Economics 18.
Principles of Accounting.

For Undergraduates and Graduates.

Course Announcement

181 hf. The Principles of Accounting. — Lectures, discussions, and reports. Half-course (first half-year). Mon., Wed., and (at the pleasure of the instructor) Fri., at 3.30. Mr. W. M. Cole.

Source: Harvard University. Announcement of the Courses of Instruction provided by the Faculty of Arts and Sciences for the Academic Year 1900-1901, p. 43.

Enrollment

181 hf. Mr. W. M. Cole. — The Principles of Accounting.

Total 56: 43 Seniors, 4 Juniors, 2 Sophomores, 7 Others.

Source: Harvard University. Annual Reports of the President and Treasurer of Harvard College, 1900-01, p. 64.

1900-01
ECONOMICS 18
[Mid-year Examination]

Problems 1 to 5 inclusive form a connected whole;
but 6
and 7 may be substituted for 4 and 5

I

  1. Construct a rough ledger (omitting rulings and index-memoranda) to correspond with the following trial-balance:
Real estate $150,000 Proprietor $244,275
Plant $60,000 Merchandise $401,000
Patents $40,000 Rent $6,000
Supplies $228,000 Bills payable $14,000
Wages $127,000 Accounts payable $43,000
Coal $9,000 Reserve fund $12,000
Insurance $4,500
Trade discounts $8,000
Interest $1,500
Bills receivable $10,000
Accounts receivable $68,000
Cash $14,275
$720,275 $720,275
  1. The above trial-balance is supposed to be taken from manufacturing books that are kept on the ordinary commercial plan, i.e., without distinctive accounts for stores, manufacturing, stock, or trading; and to construct such accounts now is supposed to be either impossible or undesirable.
    If you were required to determine profit and loss for the year which these figures cover, what questions about the business should you wish to ask before reaching your conclusions? [Give your answer in the form of questions consecutively numbered.]
  2. State what would be fairly reasonable answers to your own questions above numbering the answers to correspond with the questions; and then, assuming your answers to be the real answers show a complete statement of resources and liabilities and of profit and loss.
  3. Close for the year the ledger that you constructed indicating all balances that you have transferred to other accounts and all balances that you have carried down for the new year.
  4. From the ledger as it now stands draw off a balance sheet showing the condition of the business at the beginning of the new year, assuming that the loss or gain is carried directly to the proprietor’s account.
  5. Journalize the following:

A gives you his note for $100, bearing interest, dated a month ago.
You discount at a bank a note for $100 payable in a month
B gives you A’s note for $100 payable in one month, and buy goods for $100 on one month’s time.
Your book-keeper charged bills receivable and credited B when B paid his bill by your own note returned to you. A counter entry is to be made, so that the original wrong entry need not be erased

  1. What is the distinguishing feature of double entry? Are two postings made for every entry? If not, what devices are employed for reducing the number of postings?

II
Omit one

  1. The balance sheet of a corporation on January 1, 1899, stood as follows:
Real estate $50,000 Capital stock $200,000
Plant $95,000 Accounts payable $20,000
Horses, etc. $15,000 Bills payable $25,000
Patents $20,000 Profit and loss $15,000
Merchandise $30,000
Accounts receivable $30,000
Cash $20,000
$260,000 $260,000

On January 1, 1900, the books showed the following facts:

Real estate $55,000 Capital stock $200,000
Plant $88,000 Accounts payable $12,000
Horses, etc. $12,000 Bills payable $17,000
Patents $19,000 Profit and loss $33,000
Merchandise $42,000
Accounts receivable $28,000
Cash $18,000
$262,000 $262,000

What has become of the profits earned?

Should you recommend that a dividend be declared? State your reasons.

  1. How should you treat interest received on a bond bought above par?
  2. Describe the following, and state the distinguishing feature of each: a real account; a nominal account; a suspense account; reserve fund: a sinking fund
  3. If payments are received on account of goods in process of manufacture, should such payments appear on the balance sheet? If so, where?
  4. Describe three different methods of treating depreciation, and show how each would appear upon the books. To what circumstances on a railroad is each adapted?
  5. A corporation is formed to unite and continue the business of three concerns, A, B, and C, engaged in the same industry. The books of the concerns show the following:
A B C
Assets (valuation) $80,000 $160,000 $120,000
Liabilities (external) $20,000 $80,000 $90,000
Average profit, last three years 10% 14% 30%
Average profit, preceding three years 9 17 25
Average profit, prior three years 10 20 20

On what basis should you determine the total amount of capital stock to be issued by the new corporation, and on what basis should you apportion it to these three concerns?

Source: Harvard University Archives. Harvard University Mid-year Examinations, 1852-1943. Box 4, Bound volume: Examination Papers, Mid-Years, 1900-01.

 

__________________________________

Economics 19.
General View of Insurance.

Primarily for Graduates.

Course Announcement

192 hf. A General View of Insurance. — Lectures and reports. Half-course (second half-year). Mon., Wed., and (at the pleasure of the instructor) Fri., at 3.30. Professor Wambaugh.
Course 19 cannot be counted towards the degree of A.B.

Source: Harvard University. Announcement of the Courses of Instruction provided by the Faculty of Arts and Sciences for the Academic Year 1900-1901, p. 43.

Enrollment

192 hf. Professor Wambaugh. — A General View of Insurance.

Total 9: 6 Seniors, 3 Others.

Source: Harvard University. Annual Reports of the President and Treasurer of Harvard College, 1900-01, p. 64.

1900-01
ECONOMICS 192
[Year-end Examination]

One of the paragraphs may be omitted.

  1. From the point of view of the person procuring the policy, what is the purpose of insurance?
  2. From the point of view of the community, what are the advantages and the disadvantages of insurance?
  3. Give some account of three insurance books, pamphlets, or periodicals.
  4. Tell what you know of the history of insurance.
  5. Give a classification of the provision of the New York standard form of fire insurance policy,
  6. If either party to the fire insurance contract wishes to terminate the insurance, what are his rights?
  7. What are the benefits and the dangers of fire insurance by government?
  8. Describe ordinary life policies, single payment life policies, twenty payment life policies, endowment policies, tontine policies, assessment insurance.
  9. If a person thirty years of age wishes to obtain a life insurance policy for a single premium, how is the premium calculated?
  10. What are the chief differences between fire insurance and marine insurance?
  11. Discuss any insurance topic of which you have made a special study. 

Source: Harvard University Archives. Examination Papers, 1873-1915. Box 5. Bound Volume: Examination Papers 1900-01. Papers Set for Final Examinations in History, Government, Economics, Philosophy, Education, Fine Arts, Architecture, Landscape Design, Music in Harvard College (June, 1901), p. 40.

 

__________________________________

Economics 20d.
Adam Smith and Ricardo.

Primarily for Graduates.

Course Announcement

20d. Adam Smith and Ricardo. Half-course. Professor Taussig.

Source: Harvard University. Announcement of the Courses of Instruction provided by the Faculty of Arts and Sciences for the Academic Year 1900-1901, p. 43.

Enrollment

20d1 hf. Professor Taussig. — Adam Smith and Ricardo.

Total 12: 7 Graduates, 5 Seniors.

Source: Harvard University. Annual Reports of the President and Treasurer of Harvard College, 1900-01, p. 64.

1900-01
ECONOMICS 20d
[Final examination]
  1. Compare Ricardo’s conclusions with Adam Smith’s on the course of wages, profits, and rent, as society advances: discussing not only the conclusions themselves, but the reasoning by which the two writers arrive at them.
  2. Under what circumstances are real wages high, according to Adam Smith? according to Ricardo?
  3. Adam Smith’s doctrine on labor as a measure of value; Ricardo’s strictures thereon; and Ricardo’s own doctrine.
  4. S. Mill in his Autobiography says that “it was one of my father’s main objects to make me apply to Smith’s more superficial view of political economy the superior lights of Ricardo, and to detect what was fallacious in Smith’s arguments or erroneous in his conclusions.” Set forth how you believe the two Mills (father and son) set about this task as to Adam Smith’s reasoning on the following topics:—
    1. the mode in which the payment of heavy foreign obligations is brought about by the exportation of goods, not by the outflow of specie;
    2. the distinction between that land which always affords rent, and that which sometimes does and sometimes does not;
    3. the effect of foreign trade in raising the general rate of profits in a country.
  5. “That able but wrong-headed man, David Ricardo; shunted the car of Economic science on to a wrong line, a line, however, on which it was further urged by his equally able and wrong-headed admirer, John Stuart Mill.” — W. S. Jevons.
    What grounds are there for assenting to this judgment? What grounds for dissenting from it?

Source: Harvard University Archives. Examination Papers, 1873-1915. Box 5. Bound Volume: Examination Papers 1900-01. Papers Set for Final Examinations in History, Government, Economics, Philosophy, Education, Fine Arts, Architecture, Landscape Design, Music in Harvard College (June, 1901), pp. 40-41.

Image Source: Detail from cover of the Harvard Class Album 1946.

Categories
AEA Bibliography

American Economic Association. Monographs: 1886-1896

 

Besides transcribing and curating archival content for Economics in the Rear-view Mirror, I occasionally put together collections of links to books and other items of interest on pages or posts that constitute my “personal” virtual economics reference library. In this post you will find links to early monographs/papers published by the American Economic Association. 

Links to the contents of the four volumes of AEA Economic Studies, 1896-1899 have also been posted.

A few other useful collections:

The virtual rare-book reading room (classic works of economics up to 1900)

The Twentieth Century Economics Library

Laughlin’s recommended teacher’s library of economics (1887)

_____________________

PUBLICATIONS OF THE AMERICAN ECONOMIC ASSOCIATION. MONOGRAPHS.
1886-1896

_____________________

General Contents and Index to Volumes I-XI.
Source: Publications of the American Economic Association, Vol XI (1896). Price 25 cents.

VOLUME I

No. 1 (Mar. 1886). Report of the Organization of the American Economic Association. By Richard T. Ely, Ph.D., Secretary. Price 50 cents.

Nos. 2 and 3 (May-Jul. 1886). The Relation of the Modern Municipality to the Gas Supply. By Edmund J. James, Ph.D. Price 75 cents.

No. 4 (Sep. 1886). Co-öperation in a Western City. By Albert Shaw, Ph.D. Price 75 cents.

No. 5 (Nov. 1886). Co-öperation in New England. By Edward W. Bemis, Ph.D. Price 75 cents.

No. 6 (Jan. 1887). Relation of the State to Industrial Action. By Henry C. Adams, Ph.D. Price 75 cents.

 

VOLUME II

No. 1 (Mar. 1887). Three Phases of Co-öperation in the West. By Amos G. Warner, Ph.D. Price 75 cents.

No. 2 (May 1887). Historical Sketch of the Finances of Pennsylvania. By T. K. Worthington, Ph.D. Price 75 cents.

No. 3 (Jul. 1887). The Railway Question. By Edmund J. James, Ph.D. Price 75 cents.

No. 4 (Sep. 1887). The Early History of the English Woolen Industry. By William J. Ashley, M.A. Price 75 cents.

No. 5 (Nov. 1887). Two Chapters on the Mediaeval Guilds of England. By Edwin R. A. Seligman, Ph.D. Price 75 cents.

No. 6 (Jan. 1888). The Relation of Modern Municipalities to Quasi-Public Works. By H. C. Adams, George W. Knight, Davis R. Dewey, Charles Moore, Frank J. Goodnow and Arthur Yager. Price 75 cents.

 

VOLUME III

No. 1 (Mar. 1888). Three Papers Read at Meeting in Boston: “The Study of Statistics in Colleges,” by Carroll D. Wright; “The Sociological Character of Political Economy,” by Franklyn H. Giddings; “Some Considerations on the Legal-Tender Decisions,” by Edmund J. James. Price 75 cents.

No. 2 (May 1888). Capital and its Earnings. By John B. Clark, A.M. Price 75 cents.

No. 3 (Jul. 1888) consists of three parts: “Efforts of the Manual Laboring Class to Better Their Condition,” by Francis A. Walker; “Mine Labor in the Hocking Valley,” by Edward W. Bemis, Ph.D.; “Report of the Second Annual Meeting,” by Richard T. Ely, Secretary. Price 75 cents.

Nos. 4 and 5 (Sep.-Nov. 1888). Statistics and Economics. By Richmond Mayo-Smith, A.M. Price $1.00.

No. 6 (Jan. 1889). The Stability of Prices. By Simon N. Patten, Ph.D. Price 75 cents.

 

VOLUME IV

No. 1 (Mar. 1889). Contributions to the Wages Question: “The Theory of Wages,” by Stuart Wood, Ph.D.; “The Possibility of a Scientific Law of Wages,” by John B. Clark, A.M. Price 75 cents.

No. 2 (Apr. 1889). Socialism in England. By Sidney Webb, LL.B. Price 75 cents.

No. 3 (May. 1889). Road Legislation for the American State. By Jeremiah W. Jenks, Ph.D. Price 75 cents.

No. 4 (Jul. 1889). Report of the Proceedings of Third Annual Meeting of the American Economic Association, by Richard T. Ely, Secretary; with addresses by Dr. William Pepper and Francis A. Walker. Price 75 cents.

No. 5 (Sep. 1889). Three Papers Read at Third Annual Meeting: “Malthus and Ricardo,” by Simon N. Patten; “The Study of Statistics,” by Davis R. Dewey, and “Analysis in Political Economy,” by William W. Folwell. Price 75 cents.

No. 6 (Nov. 1889). An Honest Dollar. By E. Benjamin Andrews. Price 75 cents.

 

VOLUME V

No. 1 (Jan. 1890). The Industrial Transition in Japan. By Yeijiro Ono, Ph.D. Price $1.00.

No. 2 (Mar. 1890). Two Prize Essays on Child-Labor: I. “Child Labor,” by William F. Willoughby, Ph.D.; II. “Child Labor,” by Miss Clare de Graffenried. Price 75 cents.

Nos. 3 and 4 (May-Jul. 1890). Two Papers on the Canal Question. I. By Edmund J. James, Ph.D.; II. By Lewis M. Haupt, A.M., C.E. Price $1.00.

No. 5 (Sep. 1890). History of the New York Property Tax. By John Christopher Schwab, A.M. Ph.D. Price $1.00.

No. 6 (Nov. 1890). The Educational Value of Political Economy. By Simon N. Patten, Ph.D. Price 75 cents.

 

VOLUME VI

No. 1 and 2 (Jan.-Mar. 1891). Report of the Proceedings of the Fourth Annual Meeting of the American Economic Association. Price $1.00.

No. 3 (May 1891). I. “Government Forestry Abroad,” by Gifford Pinchot; II. “The Present Condition of the Forests on the Public Lands,” by Edward A. Bowers; III. “Practicability of an American Forest Administration,” by B. E. Fernow. Price 75 cents.

Nos. 4 and 5 (Jul.-Sep. 1891). Municipal Ownership of Gas in the United States. By Edward W. Bemis, Ph.D. with appendix by W. S. Outerbridge, Jr. Price $1.00.

No. 6 (Nov. 1891). State Railroad Commissions and How They May be Made Effective. By Frederick C. Clark, Ph.D. Price 75 cents.

 

VOLUME VII

No. 1 (Jan. 1892). The Silver Situation in the United States. Ph.D. By Frank W. Taussig, LL.B., Ph.D. Price 75 cents.

Nos. 2 and 3 (Mar.-May 1892). On the Shifting and Incidence of Taxation. By Edwin R.A. Seligman, Ph.D. Price $1.00.

Nos. 4 and 5 (Jul.-Sep. 1892). Sinking Funds. By Edward A. Ross, Ph.D. Price $1.00.

No. 6 (Nov. 1892). The Reciprocity Treaty with Canada of 1854. By Frederick E. Haynes, Ph.D. Price 75 cents.

 

VOLUME VIII

No. 1 (Jan. 1893). Report of the Proceedings of the Fifth Annual Meeting of the American Economic Association. Price 75 cents.

Nos. 2 and 3 (Mar.-May 1893). The Housing of the Poor in American Cities. By Marcus T. Reynolds, Ph.B., M.A. Price $1.00.

Nos. 4 and 5 (Jul.-Sep. 1893). Public Assistance of the Poor in France. By Emily Greene Balch, A.B. Price $1.00.

No. 6 (Nov. 1893). The First Stages of the Tariff Policy of the United States. By William Hill, A.M. Price $1.00.

 

VOLUME IX

No. 1 (Supplement, Jan. 1894). Hand-Book and Report of the Sixth Annual Meeting. Price 50 cents.

Nos. 1 and 2 (Jan.-Mar. 1894). Progressive Taxation in Theory and Practice. By Edwin R.A. Seligman, Ph.D. Price $1.00, cloth $1.50.

No. 3 (May. 1894). The Theory of Transportation. By Charles H. Cooley Price 75 cents.

No. 4 (Aug. 1894). Sir William Petty. A Study in English Economic Literature. By Wilson Lloyd Bevan, M.A., Ph.D. Price 75 cents.

Nos. 5 and 6 (Oct.-Dec. 1894). Papers Read at the Seventh Annual Meeting: “The Modern Appeal to Legal Forces in Economic Life,” (President’s annual address) by John B. Clark, Ph.D.; “The Chicago Strike”, by Carroll D. Wright, LL.D.; “Irregularity of Employment,” by Davis R. Dewey, Ph.D.; “The Papal Encyclical Upon the Labor Question,” by John Graham Brooks; “Population and Capital,” by Arthur T. Hadley, M.A. Price $1.00.

 

VOLUME X

No. 3, Supplement, (Jan. 1895). Hand-Book and Report of the Seventh Annual Meeting. Price 50 cents.

Nos. 1,2 and 3 (Jan.-Mar.-May 1895). The Canadian Banking System, 1817-1890. By Roeliff Morton Breckenridge, Ph.D. Price $1.50; cloth $2.50.

No. 4 (Jul. 1895). Poor Laws of Massachusetts and New York. By John Cummings, Ph.D. Price 75 cents.

Nos. 5 and 6 (Sep.-Nov. 1895). Letters of Ricardo to McCulloch, 1816-1823. Edited, with introduction and annotations by Jacob H. Hollander, Ph.D. Price $1.25; cloth $2.00.

 

VOLUME XI

Nos. 1, 2 and 3 (Jan.-Mar.-May 1896). Race Traits and Tendencies of the American Negro. By Frederick L. Hoffman, F.S.S., Price $1.25; cloth $2.00.

No. 4 (Jul. 1896). Appreciation and Interest. By Irving Fisher, Ph.D., Price 75 cents.

 

Image Source: As of 1909 the former Presidents of the American Economic Association (S. N. Patten in the center, then clockwise from upper left are R. T. Ely, J. B. Clark, J. W. Jenks, F. W. Taussig.) in Reuben G. Thwaites “A Notable Gathering of Scholars,” The Independent, Vol. 68, January 6, 1910, pp. 7-14.

Categories
Columbia Economists Pennsylvania

Pennsylvania. Economics Ph.D. alumnus. Columbia professor Henry R. Seager, 1894

Another post in the irregular series “Meet an economics Ph.D.” Henry Rogers Seager’s education and career took him from Ann Arbor (University of Michigan) through Baltimore (Johns Hopkins University), Germany/Austria (Halle, Berlin, Vienna), and Philadelphia (University of Pennsylvania before ending up in New York City (Columbia). 

______________________

Earlier posts for Henry Rogers Seager
at Economics in the Rear-View Mirror:

List of papers published as of Seager’s appointment by Columbia in 1902.

Syllabus for “The Trust Problem”, 1907.

Published Lecture on Economics, 1907-08.

Memorial minute, 1930

______________________

BIOGRAPHICAL NOTE

Seager, Henry Rogers (July 21, 1870-Aug. 23, 1930), economist, was born in Lansing, Michigan, the son of Schuyler Fiske Seager, a lawyer, and Alice (Berry) Seager. Graduating at the University of Michigan in 1890, he did further work during the succeeding years at the Johns Hopkins University, at the Universities of Halle, Berlin, and Vienna, and at the University of Pennsylvania, where he received the Ph.D. degree in 1894. That year he was appointed instructor in economics in the Wharton School of Finance and Commerce, and in 1896 he was made an assistant professor; in 1902 he became adjunct professor and in 1905, professor, in Columbia University, where he served till death. On June 5, 1899, he was married to Harriet Henderson of Philadelphia who died in 1928; their son survived him.

Seager’s training as an economist was in English classicism, in the German historical method and in the peculiar Austrian approach. His published work shows clearly the influence of each. His greatest admiration was for Simon N. Patten (q.v.), with whom he served at the University of Pennsylvania but whose influence on his thought was slight. Seager’s mind was orderly and compressive rather than brilliant and generalizing; conservatism was perhaps its distinguishing characteristic. He was solid and patient, slow to conclude, and even slower to write his conclusions. One result of this was that he was less a writing scholar than one who worked with students. Literally hundreds of dissertations passed through his careful hands at Columbia and many generations of students heard his lectures on labor and on corporation problems. Always active in meliorative activities, he assisted materially in the establishment of a system of workmen’s compensation in New York; he was a supporter of the Survey (formerly Charities and the Commons) and for many years a member of its board of directors. During the war he served as secretary of the Shipbuilding Labor Adjustment Board, and in 1919-20, he was executive secretary of the President’s Second Industrial Conference. He was one of the founders and three times president of the American Association for Labor Legislation. He was frequently consulted by philanthropists, legislators and publicists; he was a member of the editorial board) of The Political Science Quarterly, and in 1922 was president of the American Economic Association. In all these varied activities he had one purpose: to better social conditions within the framework of laissez-faire. He possessed a determined faith that this could be done and worked constantly to show the way. Melioration consisted in making changes here and there, which while not disturbing fundamental arrangements, reduced their burden on less favored individuals. Improvement consisted in legal change and a large part of his effort was always directed toward reform by legislation.

His most considerable work is Principles of Economics, first published under this title in 1913, which grew out of his Introduction to Economics (1904 and later editions [3rd edition, 1910]) and appeared in its final form [3rd edition] in 1923. The most important of his other writings are Trust and Corporation Problems (1929), with C.A. Gulick, Jr. and the posthumous volume, Labor and Other Economic Essays (1931), to which is attached his complete bibliography. Somewhat more than the final half of the Principles of Economics is devoted to essays on important problems: banking, the tariff, railroads, trusts, taxation, labor and social insurance. The theoretical section begins with a consideration of consumption, progresses through value and production, and ends with distribution. There were many books published during this period with much the same outline; but Seager’s was characterized by emphasis on all that pertained to human welfare. This led to stress on consumption and on the demand side of the value equilibrium, as well as to extra consideration of monopoly gains. The discussion of distribution was carried out within the framework of the “specific productivity” analysis but with more than usual weight given to such subjective influences as the balancing, in consumers’ and producers’ minds, of marginal disutilities over against marginal utilities. The conclusions were usually optimistic. Seager believed in progress and believed that, under the going system, it was being achieved. He felt, for instance, that capital goods were multiplying more rapidly than population and that this would tend to raise standards of living. He did not believe, however, that the possibilities of progress which inhere in the system insure automatic betterment. Groups of interested people, with journals and propaganda, need to be vigilant in the public interest. This duty of the good citizen, as Seager saw it, was best exemplified in his own career. He never became aware of a duty that he did not forthwith perform. In his posthumous Labor and Other Economic Essays his program is outlined: “The two great objects to be aimed at are: 1. To protect wage-earners in the continued enjoyment of standards of living to which they are already accustomed. II. To assist them to attain to higher standards of living” (p.131). The contingencies which were the principal threats to existing standards were “(1) industrial accidents, (2) illness, (3) invalidity and old age, (4) premature death, (5) unemployment” (ibid.) All these, Seager felt, were legitimate objects of collective action. As for raising standards, this was largely dependent on industrial advance and on better education.

To all persons of Seager’s generation the rather the sudden rise of a complete alternative system in Russia offered a shock to which adjustment was necessarily slow. Because everything there was so antithetical to the system to which so many theoretical hostages had been given, the immediate impulse was to belittle Soviet accomplishments. Seager was exposed to the full force of the new ideas. Gradually they gained weight in his mind until at last his essential honesty compelled, not acceptance, but exploration. In 1930, with a group of companions, he undertook a journey to the scene of these new economic adventures, in the midst of which he was taken ill. He died in Kiev of pneumonia, August 23, 1930. He was thus lost to the world at the close of an old period and the beginning of a new one. His identification with economy of the opening decades of the nineteenth century was a fortuitous one, but his progress into the new years cannot be said to have fairly started. He remains an economist of laissez-faire, of more than usual significance in foreshadowing the ameliorative program which so soon became a center of Interest.

Source: Cornell University Kheel Center for Labor-Management Documentation and Archives. Henry R. Seager Research Notes and Monographs (Collection Number: 5249).

Image Source: From a 1915 portrait of Henry Rogers Seager at Wikiwand. Includes a survey of his books.

Categories
Cambridge Chicago Columbia Economists Germany Harvard History of Economics Johns Hopkins LSE Oxford Teaching Undergraduate Wisconsin Yale

Survey of Economics Education. Colleges and Universities (Seligman), Schools (Sullivan), 1911

 

In V. Orval Watt’s papers at the Hoover Institution archives (Box 8) one finds notes from his Harvard graduate economics courses (early 1920s). There I found the bibliographic reference to the article transcribed below. The first two parts of this encyclopedia entry were written by Columbia’s E.R.A. Seligman who briefly sketched the history of economics and then presented a survey of the development of economics education at  colleges and universities in Europe and the United States. Appended to Seligman’s contribution was a much shorter discussion of economics education in the high schools of the United States by the high-school principal,  James Sullivan, Ph.D.

_________________________

 

ECONOMICS
History 

Edwin R. A. Seligman, Ph.D., LL.D.
Professor of Political Economy, Columbia University

The science now known as Economics was for a long time called Political Economy. This term is due to a Frenchman — Montchrétien, Sieur de Watteville — who wrote in 1615 a book with that title, employing a term which had been used in a slightly different sense by Aristotle. During the Middle Ages economic questions were regarded very largely from the moral and theological point of view, so that the discussions of the day were directed rather to a consideration of what ought to be, than of what is.

The revolution of prices in the sixteenth century and the growth of capital led to great economic changes, which brought into the foreground, as of fundamental importance, questions of commerce and industry. Above all, the breakdown of the feudal system and the formation of national states emphasized the considerations of national wealth and laid stress on the possibility of governmental action in furthering national interests. This led to a discussion of economic problems on a somewhat broader scale, — a discussion now carried on, not by theologians and canonists, but by practical business men and by philosophers interested in the newer political and social questions. The emphasis laid upon the action of the State also explains the name Political Economy. Most of the discussions, however, turned on the analysis of particular problems, and what was slowly built up was a body of practical precepts rather than of theoretic principles, although, of course, both the rules of action and the legislation which embodied them rested at bottom on theories which were not yet adequately formulated.

The origin of the modern science of economics, which may be traced back to the third quarter of the eighteenth century, is due to three fundamental causes. In the first place, the development of capitalistic enterprise and the differentiation between the laborer and the capitalist brought into prominence the various shares in distribution, notably the wages of the laborer, the profits of the capitalist, and the rent of the landowner. The attempt to analyze the meaning of these different shares and their relation to national wealth was the chief concern of the body of thinkers in France known as Physiocrats, who also called themselves Philosophes-Économistes, or simply Économistes, of whom the court physician of Louis XVI, Quesnay, was the head, and who published their books in 1757-1780.

The second step in the evolution of economic science was taken by Adam Smith (q.v.). In the chair of philosophy at the University of Glasgow, to which Adam Smith was appointed in 1754, and in which he succeeded Hutcheson, it was customary to lecture on natural law in some of its applications to politics. Gradually, with the emergence of the more important economic problems, the same attempt to find an underlying natural explanation for existing phenomena was extended to the sphere of industry and trade; and during the early sixties Adam Smith discussed these problems before his classes under the head of “police.” Finally, after a sojourn in France and an acquaintance with the French ideas, Adam Smith developed his general doctrines in his immortal work. The Wealth of Nations, published in 1776. When the industrial revolution, which was just beginning as Adam Smith wrote, had made its influence felt in the early decades of the nineteenth century, Ricardo attempted to give the first thorough analysis of our modern factory system of industrial life, and this completed the framework of the structure of economic science which is now being gradually filled out.

The third element in the formation of modern economics was the need of elaborating an administrative system in managing the government property of the smaller German and Italian rulers, toward the end of the eighteenth century. This was the period of the so-called police state when the government conducted many enterprises which are now left in private hands. In some of the German principalities, for instance, the management of the government lands, mines, industries, etc., was assigned to groups of officials known as chambers. In their endeavor to elaborate proper methods of administration these chamber officials and their advisors gradually worked out a system of principles to explain the administrative rules. The books written, as well as the teaching chairs founded, to expound these principles came under the designation of the Chamber sciences (Camiralia or Cameral-Wissenschaften) — a term still employed to-day at the University of Heidelberg. As Adam Smith’s work became known in Germany and Italy by translations, the chamber sciences gradually merged into the science of political economy.

Finally, with the development of the last few decades, which has relegated to the background the administrative and political side of the discipline, and has brought forward the purely scientific character of the subject, the term Political Economy has gradually given way to Economics.

Development of Economic Teaching

Edwin R. A. Seligman, Ph.D., LL.D.
Professor of Political Economy, Columbia University

Europe —

As has been intimated in the preceding section, the first attempts to teach what we to-day would call economics were found in the European universities which taught natural law, and in some of the Continental countries where the chamber sciences were pursued. The first independent chairs of political economy were those of Naples in 1753, of which the first incumbent was (Genovesi, and the professorship of cameral science at Vienna in 1763, of which the first incumbent was Sonnenfels. It was not, however, until the nineteenth century that political economy was generally introduced as a university discipline. When the new University of Berlin was created in 1810, provision was made for teaching in economics, and this gradually spread to the other German universities. In France a chair of economics was established in 1830 in the Collège de France, and later on in some of the technical schools; but economics did not become a part of the regular university curriculum until the close of the seventies, when chairs of political economy were created in the faculties of law, and not, as was customary in the other Continental countries, in the faculties of philosophy. In England the first professorship of political economy was that instituted in 1805 at Haileybury College, which trained the students for the East India service. The first incumbent of this chair was Malthus. At University College, London, a chair of economics was established in 1828, with McCulloch as the first incumbent; and at Dublin a chair was founded in Trinity College in 1832 by Archbishop Whately; at Oxford a professorship was established in 1825, with Nassau W. Senior as the first incumbent. His successors were Richard Whately (1830), W. F. Lloyd (1836), H. Merivale (1838), Travers Twiss (1842), Senior (1847), G. K. Richards (1852), Charles Neate (1857), Thorold Rogers (1862), Bonamy Price (1868), Thorold Rogers (1888). and F. Y. Edgeworth (1891). At Cambridge the professorship dates from 1863, the first incumbent being Henry Fawcett, who was followed by Alfred Marshall in 1884 and by A. C. Pigou in 1908. In all these places, however, comparatively little attention was paid at first to the teaching of economics, and it was not until the close of the nineteenth century and the beginning of the twentieth that any marked progress was made, although the professorship at King’s College, London, dates back to 1859, and that at the University of Edinburgh to 1871. Toward the close of the nineteenth century, chairs in economics were created in the provincial universities, especially at Birmingham, Manchester, Liverpool, Sheffield, Bristol, Durham, and the like, as well as in Scotland and Wales; and a great impetus to the teaching of economics was given by the foundation, in 1895, of the London School of Economics, which has recently been made a part of the University of London.

— United States 

Economics was taught at first in the United States, as in England, by incumbents of the chair of philosophy; but no especial attention was paid to the study, and no differentiation of the subject matter was made. The first professorship in the title of which the subject is distinctively mentioned was that instituted at Columbia College, New York, where John McVickar, who had previously lectured on the subject under the head of philosophy, was made professor of moral philosophy and political economy in 1819. In order to commemorate this fact, Columbia University established some years ago the McVickar professorship of political economy. The second professorship in the United States was instituted at South Carolina College, Columbia, S. C, where Thomas Cooper, professor of chemistry, had the subject of political economy added to the title of his chair in 1826. A professorship of similar sectional influence was that in political economy, history, and metaphysics filled in the College of William and Mary in 1827, by Thomas Roderick Dew (1802-1846). The separate professorships of political economy, however, did not come until after the Civil War. Harvard established a professorship of political economy in 1871; Yale in 1872; and Johns Hopkins in 1876.

The real development of economic teaching on a large scale began at the close of the seventies and during the early eighties. The newer problems bequeathed to the country by the Civil War were primarily economic in character. The rapid growth of industrial capitalism brought to the front a multitude of questions, whereas before the war well-nigh the only economic problems had been those of free trade and of banking, which were treated primarily from the point of view of partisan politics. The newer problems that confronted the country led to the exodus of a number of young men to Germany, and with their return at the end of the seventies and beginning of the eighties, chairs were rapidly multiplied in all the larger universities. Among these younger men were Patten and James, who went to the University of Pennsylvania; Clark, of Amherst and later of Columbia; Farnam and Hadley of Yale; Taussig of Harvard; H. C. Adams of Michigan; Mayo-Smith and Seligman of Columbia; and Ely of Johns Hopkins. The teaching of economics on a university basis at Johns Hopkins under General Francis A. Walker helped to create a group of younger scholars who soon filled the chairs of economics throughout the country. In 1879 the School of Political Science at Columbia was inaugurated on a university basis, and did its share in training the future teachers of the country. Gradually the teaching force was increased in all the larger universities, and chairs were started in the colleges throughout the length and breadth of the land.

At the present time, most of the several hundred colleges in the United States offer instruction in the subject, and each of the larger institutions has a staff of instructors devoted to it. At institutions like Columbia, Harvard, Yale, Chicago, and Wisconsin there are from six to ten professors of economics and social science, together with a corps of lecturers, instructors, and tutors.

Teaching of Economics in the American Universities. — The present-day problems of the teaching of economics in higher institutions of learning are seriously affected by the transition stage through which these institutions are passing. In the old American college, when economics was introduced it was taught as a part of the curriculum designed to instill general culture. As the graduate courses were added, the more distinctly professional and technical phases of the subject were naturally emphasized. As a consequence, both the content of the course and the method employed tended to differentiate. But the unequal development of our various institutions has brought great unclearness into the whole pedagogical problem. Even the nomenclature is uncertain. In one sense graduate courses may be opposed to undergraduate courses; and if the undergraduate courses are called the college courses, then the graduate courses should be called the university courses. The term “university,” however, is coming more and more, in America at least, to be applied to the entire complex of the institutional activities, and the college proper or undergraduate department is considered a part of the university. Furthermore, if by university courses as opposed to college courses we mean advanced, professional, or technical courses, a difficulty arises from the fact that the latter year or years of the college course are tending to become advanced or professional in character. Some institutions have introduced the combined course, that is, a combination of so-called college and professional courses; other institutions permit students to secure their baccalaureate degree at the end of three or even two and a half years. In both cases, the last year of the college will then cover advanced work, although in the one case it may be called undergraduate, and in the other graduate, work.

The confusion consequent upon this unequal development has had a deleterious influence on the teaching of economics, as it has in many other subjects. In all our institutions we find a preliminary or beginners’ course in economics, and in our largest institutions we find some courses reserved expressly for advanced or graduate students. In between these, however, there is a broad field, which, in some institutions, is cultivated primarily from the point of view of graduates, in others from the point of view of undergraduates, and in most cases is declared to be open to both graduates and undergraduates. This is manifestly unfortunate. For, if the courses, are treated according to advanced or graduate methods, they do not fulfill their proper function as college studies. On the other hand, if they are treated as undergraduate courses, they are more or less unsuitable for advanced or graduate students. In almost all of the American institutions the same professors conduct both kinds of courses. In only one institution, namely, at Columbia University, is the distinction between graduate and undergraduate courses in economics at all clearly drawn, although even there not with precision. At Columbia University, of the ten professors who are conducting courses in economics and social science, one half have seats only in the graduate faculties, and do no work at all in the college or undergraduate department; but even there, these professors give a few courses, which, while frequented to an overwhelming extent by graduate students, are open to such undergraduates as may be declared to be advanced students.

It is necessary, therefore, to distinguish, in principle at least, between the undergraduate or college courses properly so-called, and the university or graduate courses. For it is everywhere conceded that at the extremes, at least, different pedagogical methods are appropriate.

The College or Undergraduate Instruction. — Almost everywhere in the American colleges there is a general or preliminary or foundation course in economics. This ordinarily occupies three hours a week for the entire year, or five hours a week for the semester, or half year, although the three-hour course in the fundamental principles occasionally continues only for a semester. The foundation of such a course is everywhere textbook work, with oral discussion, or quizzes, and frequent tests. Where the number of students is small, this method can be effectively employed; but where, as in our larger institutions, the students attending this preliminary course are numbered by the hundreds, the difficulties multiply. Various methods are employed to solve these difficulties. In some cases the class attends as a whole at a lecture which is given once a week by the professor, while at the other two weekly sessions the class is divided into small sections of from twenty to thirty, each of them in charge of an instructor who carries on the drill work. In a few instances, these sections are conducted in part by the same professor who gives the lecture, in part by other professors of equal grade. In other cases where this forms too great a drain upon the strength of the faculty, the sections are put in the hands of younger instructors or drill masters. In other cases, again, the whole class meets for lecture purposes twice a week, and the sections meet for quiz work only once a week. Finally, the instruction is sometime carried on entirely by lectures to the whole class, supplemented by numerous written tests.

While it cannot be said that any fixed method has yet been determined, there is a growing consensus of opinion that the best results can be reached by the combination of one general lecture and two quiz hours in sections. The object of the general lecture is to present a point of view from which the problems may be taken up, and to awaken a general interest in the subject among the students. The object of the section work is to drill the students thoroughly in the principles of the science; and for this purpose it is important in a subject like economics to put the sections as far as possible in the hands of skilled instructors rather than of recent graduates.

Where additional courses are offered to the Undergraduates, they deal with special subjects in the domain of economic history, statistics, and practical economics. In many such courses good textbooks are now available, and especially in the last class of subject is an attempt is being made here and there to introduce the case system as utilized in the law schools. This method is, however, attended by some difficulties, arising from the fact that the materials used so quickly become antiquated and do not have the compelling force of precedent, as is the case in law. In the ordinary college course, therefore, chief reliance must still be put upon the independent work and the fresh illustrations that are brought to the classroom by the instructor.

In some American colleges the mistake has been made of introducing into the college curriculum methods that are suitable only to the university. Prominent among these are the exclusive use of the lecture system, and the employment of the so-called seminar. This, however, only tends to confusion. On the other hand, in some of the larger colleges the classroom work is advantageously supplemented by discussions and debates in the economics club, and by practical exercises in dealing with the current economic problems as they are presented in the daily press.

In most institutions the study of economics is not begun until the sophomore or the junior year, it being deemed desirable to have a certain maturity of judgment and a certain preparation in history and logic. In some instances, however, the study of economics is undertaken at the very beginning of the college course, with the resulting difficulty of inadequately distinguishing between graduate and undergraduate work.

Another pedagogical question which has given rise to some difficulty is the sequence of courses. Since the historical method in economics became prominent, it is everywhere recognized that some training in the historical development of economic institutions is necessary to a comprehension of existing facts. We can know what is very much better by grasping what has been and how it has come to be. The point of difference, however, is as to whether the elementary course in the principles should come first and be supplemented by a course in economic history, or whether, on the contrary, the course in economic history should precede that in the principles. Some institutions follow one method, others the second; and there are good arguments on both sides. It is the belief of the writer, founded on a long experience, that on the whole the best results can be reached by giving as introductory to the study of economic principles a short survey of the leading points of economic history. In a few of the modem textbooks this plan is intentionally followed. Taking it all in all, it may be said that college instruction in economics is now not only exceedingly widespread in the United States, but continually improving in character and methods.

University or Graduate Instruction. — The university courses in economics are designed primarily for those who either wish to prepare themselves for the teaching of economics or who desire such technical training in methods or such an intimate acquaintance with the more developed matter as is usually required by advanced or professional students in any discipline. The university courses in the larger American institutions which now take up every important subject in the discipline, and which are conducted by a corps of professors, comprise three elements: first, the lectures of the professor; second, the seminar or periodical meeting between the professor and a group of advanced students; third, the economics club, or meeting of the students without the professor.

(1) The Lectures: In the university lectures the method is different from that in the college courses. The object is not to discipline the student, but to give him an opportunity of coming into contact with the leaders of thought and with the latest results of scientific advance on the subject. Thus no roll of attendance is called, and no quizzes are enforced and no periodical tests of scholarship are expected. In the case of candidates for the Ph.D. degree, for instance, there is usually no examination until the final oral examination, when the student is expected to display a proper acquaintance with the whole subject. The lectures, moreover, do not attempt to present the subject in a dogmatic way, as is more or less necessary in the college courses, but, on the contrary, are designed to present primarily the unsettled problems and to stimulate the students to independent thinking. The university lecture, in short, is expected to give to the student what cannot be found in the books on the subject.

(2) The Seminar: Even with the best of will, however, the necessary limitations prevent the lecturer from going into the minute details of the subject. In order to provide opportunity for this, as well as for a systematic training of the advanced students in the method of attacking this problem, periodical meetings between the professor and the students have now become customary under the name of the seminar, introduced from Germany. In most of our advanced universities the seminar is restricted to those students who are candidates for the degree of Doctor of Philosophy, although in some cases a preliminary seminar is arranged for graduate students who are candidates for the degree of Master of Arts. Almost everywhere a reading knowledge of French and German is required. In the United States, as on the European continent generally, there are minor variations in the conduct of the seminar. Some professors restrict the attendance to a small group of most advanced students, of from fifteen to twenty-five; others virtually take in all those who apply. Manifestly the personal contact and the “give and take,” which are so important a feature of the seminar, become more difficult as the numbers increase. Again, in some institutions each professor has a seminar of his own; but this is possible only where the number of graduate students is large. In other cases the seminar consists of the students meeting with a whole group of professors. While this has a certain advantage of its own, it labors under the serious difficulty that the individual professor is not able to impress his own ideas and his own personality so effectively on the students; and in our modern universities students are coming more and more to attend the institution for the sake of some one man with whom they wish to study. Finally, the method of conducting the seminar differs in that in some cases only one general subject is assigned to the members for the whole term, each session being taken up by discussion of a different phase of the general subject. In other cases a new subject is taken up at every meeting of the seminar. The advantage of the latter method is to permit a greater range of topics, and to enable each student to report on the topic in which he is especially interested, and which, perhaps, he may be taking up for his doctor’s dissertation. The advantage of the former method is that it enables the seminar to enter into the more minute details of the general subject, and thus to emphasize with more precision the methods of work. The best plan would seem to be to devote half the year to the former method, and half the year to the latter method.

In certain branches of the subject, as, for instance, statistics, the seminar becomes a laboratory exercise. In the largest universities the statistical laboratory is equipped with all manner of mechanical devices, and the practical exercises take up a considerable part of the time. The statistical laboratories are especially designed to train the advanced student in the methods of handling statistical material.

(3) The Economics Club: The lecture work and the seminar are now frequently supplemented by the economics club, a more informal meeting of the advanced students, where they are free from the constraint that is necessarily present in the seminar, and where they have a chance to debate, perhaps more unreservedly, some of the topics taken up in the lectures and in the seminar, and especially the points where some of the students dissent from the lecturer. Reports on the latest periodical literature are sometimes made in the seminar and sometimes in the economics club; and the club also provides an opportunity for inviting distinguished outsiders in the various subjects. In one way or another, the economics club serves as a useful supplement to the lectures and the seminar, and is now found in almost all the leading universities.

In reviewing the whole subject we may say that the teaching of economics in American institutions has never been in so satisfactory condition as at present. Both the instructors and the students are everywhere increasing in numbers; and the growing recognition of the fact that law and politics are so closely interrelated with, and so largely based on, economics, has led to a remarkable increase in the interest taken in the subject and in the facilities for instruction.


Economics
— In the Schools 

James Sullivan, Ph.D., Principal of Boys’ High School, Brooklyn, N.Y.

This subject has been defined as the study of that which pertains to the satisfaction of man’s material needs, — the production, preservation, and distribution of wealth. As such it would seem fundamental that the study of economics should find a place in those institutions which prepare children to become citizens, — the elementary and high schools. Some of the truths of economics are so simple that even the youngest of school children may be taught to understand them. As a school study, however, economics up to the present time has made far less headway than civics (q.v.). Its introduction as a study even in the colleges was so gradual and so retarded that it could scarcely be expected that educators would favor its introduction in the high schools.

Previous to the appearance, in 1894, of the Report of the Committee of Ten of the National Educational Association on Secondary Education, there had been much discussion on the educational value of the study of economics. In that year Professor Patten had written a paper on Economics in Elementary Schools, not as a plea for its study there, but as an attempt to show how the ethical value of the subject could be made use of by teachers. The Report, however, came out emphatically against formal instruction in political economy in the secondary school, and recommended “that, in connection particularly with United States history, civil government, and commercial geography instruction be given in those economic topics, a knowledge of which is essential to the understanding of our economic life and development” (pp. 181-183). This view met with the disapproval of many teachers. In 1895 President Thwing of Western Reserve University, in an address before the National Educational Association on The Teaching of Political Economy in the Secondary Schools, maintained that the subject could easily be made intelligible to the young. Articles or addresses of similar import followed by Commons (1895), James (1897), Haynes (1897), Stewart (1898), and Taussig (1899). Occasionally a voice was raised against its formal study in the high schools. In the School Review for January, 1898, Professor Dixon of Dartmouth said that its teaching in the secondary schools was “unsatisfactory and unwise.” On the other hand, Professor Stewart of the Central Manual Training School of Philadelphia, in an address in April, 1898, declared the Report of the Committee of Ten “decidedly reactionary,” and prophesied that political economy as a study would he put to the front in the high school. In 1899 Professor Clow of the Oshkosh State Normal School published an exhaustive study of the subject of Economics as a School Study, going into the questions of its educational value, its place in the schools, the forms of the study, and the methods of teaching. His researches serve to show that the subject was more commonly taught in the high schools of the Middle West than in the East. (Compare with the article on Civics.)

Since the publication of his work the subject of economics has gradually made its appearance in the curricula of many Eastern high schools. It has been made an elective subject of examination for graduation from high schools by the Regents of New York State, and for admission to college by Harvard University. Its position as an elective study, however, has not led many students to take it except in commercial high schools, because in general it may not be used for admission to the colleges.

Its great educational value, its close touch with the pupils’ everyday life, and the possibility of teaching it to pupils of high school age are now generally recognized. A series of articles in the National Educational Association’s Proceedings for 1901, by Spiers, Gunton, Halleck, and Vincent bear witness to this. The October, 1910, meeting of the New England History Teachers’ Association was entirely devoted to a discussion of the Teaching of Economics in Secondary Schools, and Professors Taussig and Haynes reiterated views already expressed. Representatives of the recently developed commercial and trade schools expressed themselves in its favor.

Suitable textbooks in the subject for secondary schools have not kept pace with its spread in the schools. Laughlin, Macvane, and Walker published books somewhat simply expressed; but later texts have been too collegiate in character. There is still needed a text written with the secondary school student constantly in mind, and preferably by an author who has been dealing with students of secondary school age. The methods of teaching, mutatis mutandis, have been much the same as those pursued in civics (q.v.). The mere cramming of the text found in the poorest schools gives way in the best schools to a study and observation of actual conditions in the world of to-day. In the latter schools the teacher has been well trained in the subject, whereas in the former it is given over only too frequently to teachers who know little more about it than that which is in the text.

See also Commercial Education.

 

References: —

In Colleges and Universities: —

A Symposium on the Teaching of Elementary Economics. Jour. of Pol. Econ., Vol. XVIIl, June, 1910.

Cossa, L. Introduction to the Study of Political Economy: tr. by L. Dyer. (London, 1893.)

Mussey, H. R. Economies in the College Course. Educ. Rev. Vol. XL, 1910, pp. 239-249.

Second Conference on the Teaching of Economics, Proceedings. (Chicago, 1911.)

Seligman, E. R. A. The Seminarium — Its Advantages and Limitations. Convocation of the University of the State of New York, Proceedings. (1892.)

In Schools: —

Clow, F. R. Economics as a School Study, in the Economic Studies of the American Economic Association for 1899. An excellent bibliography is given. It may be supplemented by articles or addresses since 1899 which have been mentioned above. (New York, 1899.)

Haynes, John. Economics in Secondary Schools. Education, February, 1897.

 

Source: Paul Monroe (ed.), A Cyclopedia of Education, Vol. II. New York: Macmillan, pp. 387-392.

Source: E.R.A. Seligman in Universities and their Sons, Vol. 2 (1899), pp. 484-6.

 

Categories
Courses Curriculum Pennsylvania

Pennsylvania. Economics Course Offerings, 1897-98

 

The Department of Philosophy at the University of Pennsylvania in 1897/98 corresponded to what today is generally called the Graduate Faculty of Arts and Sciences. The subjects of economics, politics, sociology, and statistics constituted the tenth group of the sixteen groups making up the Department of Philosophy. A Ph.D. candidate would select a major subject from the group and two minor subjects (recommended that “at least one minor be taken outside the group in which the major lies”). The course offerings and their short descriptions for group X have been transcribed below.

___________________

Departments of the University of Pennsylvania
as of 1897/98
and the year they were established.

(1740. Charitable School; out of which grew, in)
1751. The College.
1765. The Department of Medicine.
1 790-1850. The Department of Law.
1865. The Auxiliary Department of Medicine.
1874. The University Hospital.
1878. The Department of Dentistry.
1883. The Department of Philosophy.
1884. The Department of Veterinary Medicine.
1885. The Veterinary Hospital.
1885. The Department of Physical Education.
1889. The Museum of Archaeology and Paleontology.
1891. The University Library. (Building erected)
1892. The Laboratory of Hygiene.
1892. The Wistar Institute of Anatomy and Biology.

Department of Philosophy

  1. Semitic Languages
  2. American Archaeology and Languages
  3. Indo-European Philology
  4. Classical Languages
  5. Germanic Languages
  6. Romanic Languages
  7. English
  8. Philosophy, Ethics, Psychology and Pedagogy
  9. History
  10. Economics, Politics, Sociology and Statistics
  11. Mathematics
  12. Astronomy
  13. Physics
  14. Chemistry
  15. Botany and Zoölogy
  16. Geology and Mineralogy

 

The degrees conferred in the Faculty of Philosophy are Doctor of Philosophy, (Ph. D. ), Master of Arts, (A. M.), and Master of Science, (M.S.).

___________________

Faculty: 10. Economics, Politics, Sociology and Statistics

Simon N. Patten, Ph.D., Professor of Political Economy
Roland P. Falkner, Ph.D. Associate Professor of Statistics
John Quincy Adams, Ph.D., Assistant Professor of Political Science
Emory R. Johnson, Ph.D., Assistant Professor of Transportation and Commerce.
Samuel McCune Lindsay, Ph.D., Assistant Professor of Sociology
Leo S. Rowe, Ph.D., Assistant Professor of Political Science
Henry R. Seager, Ph.D., Assistant Professor of Political Economy
James T. Young, Ph.D., Lecturer, Administration

___________________

X. ECONOMICS, POLITICS, SOCIOLOGY AND STATISTICS.

Professor Patten, Chairman; Associate Professor Falkner; Assistant Professors Adams, E. R. Johnson, Lindsay, Rowe and Seager; Dr. Young.

Majors —

Political Science, Economics, Transportation and Commerce, Statistics, Sociology.

Minors —

1. Political Science; A 1, 4; B 10.
2. Public Finance; A 3, and B 1, 2, or C 1, 2.
3. Administration; A 8, 6, 3, or A 9, 2, 4b; and C 5d.
4. Economic Theory; B 1, 2, and, B 10, or Mill’s Political Economy(2 hours a week).
5. History of Economic Thought; B 1, 2, 3, 4.
6. Economic History; B 10, and Group IX. C 2, 4.
7. Transportation and Commerce; B 11, 12, and A 4, or B 1, 2, or C 1, 2.
8. Statistics; C 1, 2, 3, 4, or C 1, 2, 3, and A3 (one term), or B 11.
9. Sociology; C 5c, 6, B 1, or C 5b, 6, 1, or C 5c 5d, A 4.

A.POLITICAL SCIENCE, FINANCE AND ADMINISTRATION.

Assistant Professor Adams.

  1. History of Modern Political Ideas (2 hours per week)

Lectures covering the period from Bodin to the present, supplemented by prescribed reading. Relation between political thought and economic and social conditions will be studied, with special reference to the recent political development of England.

  1. Readings in Political Science (1 hour per week)

Reading and discussion of a few recent works on special phases of political problems.

  1. Public Finance (2 hours per week)

Taxation; the classification, incidence and effect of the several taxes. Methods of raising federal revenue in the United States; the revenue systems of the leading States (Second Term).

Assistant Professor Rowe.

  1. The Individual and the State (2 hours per week)

(a) (First Term.) The concept of individual liberty at different periods of political development. Relation to law and government. Distinction between civil and political liberty.  Guarantees, legal and political. The idea of social liberty in democratic communities.

(b) (Second Term.) Relation of the State to industrial action. Socialism and Individualism. Influence of economic and social changes on the sphere of activity of the State. Growth of free contract. Effect of recognition by the State of new forms of association.

  1. History and Theory of the State (2 hours per week)

(a) (First Term.) The general principles of political science. Its province and problems. Relation to the social sciences. Nature of political association; origin of the State; source and function of law. (Omitted in 1897-98.)

(b) (Second Term.) The concept of the State at different epochs. Growth of the distinction between State and government. Ends of the State; sphere of its authority; forms of the State and of government. (Omitted in 1897-98.)

  1. Municipal Government and Institutions (2 hours per week)

(a) (First Term.) The city and conditions of city life in the ancient, mediaeval and modern world. The relation between city and State in Europe and the United States.

(b) (Second Term.) Problems of modern city life. Relation to quasi-public works. Social problems.

  1. The Analytical School of Jurisprudence in its Relation to Political Science. (Second Term) (2 hours per week)

Selected readings from Bentham, Austin, Holland, Maine and Pollock. Relation of this school of thought to the doctrine of natural law and natural rights. Contrast between the historical and analytical concepts in political science.

Dr. Young.

  1. Administration (1 hour per week)

Local rural administration. A comparison of the organization and methods of local administration in Europe and America. Provinz, Bezirk, Kreis; Department, Arrondissement ; County, Township, etc. Modern growth and tendencies.

  1. Practical Problems in Administration (1 hour per week)

Studies in special questions, such as the formation and growth of systems of Civil Service; the development of boards and commissions of administrative control; the administration of factory legislation.

 

B. — ECONOMIC THEORY, ECONOMIC HISTORY, AND TRANSPORTATION AND COMMERCE.

Professor Patten.

  1. History of Political Economy (First Term) (2 hours per week)

The rise and development of the classical school of economists. Adam Smith and Ricardo form the natural centre of the study. Especial attention is given to the sources on which Smith’s Wealth of Nationsis based; and, in the latter part of the course, to the interpretation of Ricardo’s writings.

  1. Recent Development of Political Economy (Second Term) (2 hours per week)

The American economists are carefully studied. Prominence is given to the recent development of the theory of rent, to the theory of marginal utility, to the new aspects of the science resulting from an emphasis of the dynamic causes of social progress, to recent attempts to substitute consumption for production as the starting point of investigation and the basis of theory.

  1. The Relation of English Philosophy to Economics in the Eighteenth Century (First Term) (2 hours per week)

Philosophy and economics have in England been intimately related. The purpose of this course is to bring out the facts, character and extent of this relation and its results on both  sides.

  1. Scope and Method of Political Economy (Second Term) (2 hours per week)

Limitations of scope in economic investigation; its causes as deducible from the history of the science. The method of this science will be compared with that of others. The processes by which the great writers established their characteristic doctrines will be particularly considered. (Omitted in 1897-98.)

  1. Practical Applications of Economic Theory (First Term) (2 hours per week)

The object is to show the practical importance of the new theories, and the changes they will work in public opinion. Among the topics considered will be: taxation, wages, money, retail prices, population, the causes of poverty, the standard of life, co-operation, trades unions, the dependent classes. The distribution of the surplus will be specially considered, along with its proper weight as a factor in practical problems. (Omitted in 1897-98.)

  1. The Problems of Sociology (Second Term) (2 hours per week)

The possibility of a science of sociology; the attempts to found such a science, and the obstacles encountered; the relation of economics to sociology.

  1. Investigation of Special Topics (2 hours per week)

To promote original work and aid in the preparation of theses, special hours are assigned for the discussion of these investigations. Each student must select a topic, and give three hours a week to it throughout the year.

Assistant Professor Seager.

  1. Economic Conference (2 hours per week)

Assigned portions of the following works will be read and discussed: Böhm-Bawerk, Positive Theory of Capital; Hadley, Economics; Smart, Studies in Economics. Reports will be made on cognate topics of special interest.

  1. Advanced Reading in German and French Economics (2 hours per week)

Roscher, Grundlagen der Nationalökonomieand Philippovich, Allgemeine Volkswirthschaftslehre.
Leroy-Beaulieu, Traité théorique et pratique d’économie politique.

  1. The Industrial History of England and the Development of Economic Theory from 1750 to 1870 (2 hours per week)

The main features of the industrial revolution will be described, and the effects of the growth of the factory system on the position of the wage-earner, on legislation and on economic thought, will be considered. Then attention will be directed to the Corn-Law agitation, growing out of the Napoleonic war period; and the results of the Free Trade policy, inaugurated in 1846, will be discussed. Topics will be assigned for special reports. Lectures and assigned reading.

Assistant Professor E. R. Johnson.

  1. Theory of Transportation (1 hour per week)

General Theory of Transportation. The economic functions and the ideal organization of transportation. Competition and co-operation in railway management. The past and present functions of traffic associations. Relation of the State to transportation. Theories underlying governmental regulation in the United States and other countries. The theory of rates. Lectures accompanied by an outline of assigned readings.

  1. American Railway Transportation (2 hours per week)

Seminary, based mainly upon documentary sources. The work comprises (1) an examination of the salient features of the organization and management of railway transportation in the United States; and (2) a special study of railway regulation by the States and by the Federal government.

  1. Transportation Systems of the United Kingdom and Germany (1 hour per week)

Organization, relation to the government, results of regulation of railways in the United Kingdom, and of State ownership in Germany. (Omitted in 1897-98.)

  1. History of Commerce since 1500 (1 hour per week)

Lectures, assigned readings and investigations. Special attention will be given to the causes that have given the United Kingdom the commercial supremacy of the world. (Omitted in 1897-98.)

 

C. — SOCIOLOGY AND STATISTICS.

Associate Professor Faulkner.

  1. Introduction to Statistics (First Term) (2 hours per week)

A study of the statistics of population will serve to develop the general aspects of the science, the organization of statistical work, and the possibilities and limitations of statistical argument. Familiarity with the literature and the sources of statistical information will be the chief aim.

  1. Statistics of Economic Problems (Second Term) (2 hours per week)

This course supplements course 1: foreign commerce, railroads, banks, coinage, prices, strikes, wages, cost of living, will be studied from a statistical point of view. The literature will be reviewed and criticised.

  1. History and Theory of Statistics (First Term) (2 hours per week)

Summary of statistical efforts and statistical theory, culminating in the elaboration of statistical methods. Consideration of statistics as a science of method; the basis and form of  statistical judgments. (Open to students who have taken courses 1 and 2.)

  1. Statistical Organization (Second Term) (2 hours per week)

The history, organization and function of statistical agencies at home and abroad will be surveyed. The factors that contribute to the efficiency of census departments, labor bureaus, etc., will be studied. The statistical offices of the United States will be studied and their methods criticised. (Open to students who have taken courses 1 and 2.)

Assistant Professor Lindsay.

  1. Theory of Sociology (Two Years) (2 hours per week)

A systematic course in four parts; each part requiring two hours per week for half a year, and constituting an independent course.

(a) Methodology of the social sciences; their relations to economics. The problems of sociology. Brief review of the leading attempts to construct sociology as a science — Aristotle, Montesquieu, Comte, Spencer, Ward and Giddings.

(b) Primitive Society. Evolution of social organization and social control among primitive men. Special study of the forms of the family, and of the psychology of primitive society. Particular attention is given to the records of the American Indians.

(c) Structure of Modern Society. A study of the forms of social organization and social control resulting from modern demogenic association. The basis and relative strength of educational, political, religious and philanthropic factors.

(d) Social Movements of Modern Times. A discussion of the leading practical social problems of to-day, with a view to establishing a consistent theoretical point of view, and a method as a guide to the study of temporary and changing social conditions.

Note. — 5 d is open only to students who have had parts a, bor c, or an equivalent.

  1. Social-Debtor Classes (2 hours per week)

This course deals with those groups and individuals in modern society who contribute nothing to the social surplus, but who participate in it, and hence are social debtors. They are considered under the heads of (1) the reclaimable class; (2) the naturally dependent class; (3) the pauper class; (4) the criminal classes. The methods of dealing with these groups are likewise classified and discussed.

  1. Sociological Field Work (2 hours per week)

For special investigation of local social conditions and institutions. Opportunities for visiting industrial, educational and philanthropic institutions are afforded. Each student is expected to select a topic and make a careful study of some concrete social problem, with special reference to conditions in Philadelphia. Meetings for private conference and discussion are assigned.

Note. — In the academic year 1896-97, a special sociological investigation of the condition of the Negroes in the Seventh Ward of Philadelphia, is being carried on under the direction of William E. Burkhardt Du Bois, A. B., Ph. D., who was appointed for the year Assistant in Sociology for this purpose. The College Settlement of Philadelphia is co-operating in this investigation, and has appointed Miss Isabel Eaton, B. L., Fellow in charge of its work.

 

SEMINARY.

Seminary in Political Science, Economics and Sociology (2 hours per week)

The seminary meets each Monday evening during the academic year, for the presentation and discussion of papers prepared by the members under the direction of the professors in charge of the several topics in Group IX.

 

Source: Catalogue of the University of Pennsylvania. Fasciculus of the Department of Philosophy. Announcements for Session 1897-98, pp. 44-50.

 

Categories
Economists Germany Pennsylvania

Pennsylvania. Short encyclopaedia article on Simon Patten, 1903

 

Today’s artifact is a sample short biography of an American economist that I found in The New International Encyclopaedia (eds.: F.M. Colby, H.T. Peck, and D.C. Gilman) that was published in New York City, 1902-04. This encyclopaedia looks like a convenient source of brief mid- and late-career assessments of the movers-and-shakers of economics at a time when their moves were still shaking (at least their students) that I shall return to from time to time.

 

For much more on the life and career of this University of Pennsylvania economist, Simon N. Patten, links can be found at the page dedicated to him at The History of Economic Thought website. Cf. Rexford G. Tugwell. “Notes on the Life and Work of Simon Nelson Patten.” Journal of Political Economy, vol. 31, no. 2, 1923, pp. 153–208.

__________________

PATTEN, Simon Nelson (1852—[1922]).

An American economist, born at Sandwich, Ill. He was educated at Jennings’s Seminary (Ill.), Northwestern University (Ill.), and at the University of Halle, Germany, and received the degree of Ph.D. in 1878. During the next ten years he taught in the public schools of Iowa and Illinois. In 1888 he was elected professor of political economy at the University of Pennsylvania. His principal works are: Premises of Political Economy (1885); The Consumption of Wealth (1889) [2ndedition, 1901]; The Economic Basis of Protection (1890); The Theory of Dynamic Economics (1892); The Theory of Social Forces (1896); Development of English Thought (1899); The Theory of Prosperity (1902); Heredity and Social Progress  (1903). Professor Patten ranks as one of the most brilliant and original of American economic writers. His chief contributions to economics are his analyses of dynamic forces in economic life, of monopoly elements in value, and of the bearing of the laws of consumption upon distribution. A large part of his work is rather sociological than economic.

 

Source:  The New International Encyclopaedia, (eds. F. M. Colby, H. T. Peck, and D. C. Gilman) New York: Dodd, Mead & Co. (1903), Vol. 13. p. 797.

Image Source:  American Society for the Extension of University Teaching. Supplement to the The University Extension Bulletin. Vol. I, No. 8. Philadelphia: May 10, 1894. Copy found in Box 2 of Franklin Henry Giddings Papers, Columbia Rare Book & Manuscript Library, Folder “Photographs”.

Categories
Bibliography Harvard Suggested Reading

Harvard. Debate Briefs on International Trade Policy, ca. 1886-96

 

Print from 1897 by J. S. Pughe in Punch. shows Uncle Sam sitting in a wooden tub labeled “Dingley Bill”, rowing with oars labeled “Monopoly” in a small pool labeled “Home Market” near a sign that states “Republican Goose Pond”. The title of the prints is “A self-evident fact” with the caption “Uncle Sam Say! I want you fellows to distinctly understand that I’m not racing with you!” Beyond the pond are several large steam ships, labeled “France, Germany, Italy, England, [and] Austria” steaming ahead of Uncle Sam. While Uncle Sam protects the home market through tariffs, European nations are expanding their global markets. (Library of Congress)

The inspiration for today’s posting comes from the announcement in late January, 2018 by U.S. President Donald J. Trump that steep tariffs would be imposed on washing machines and solar panels imported into the United States.

Below you will find transcriptions for Harvard University debating briefs on tariffs, subsidies and international trade from the last decade of the 19th century. While economics as a science has shown some considerable progress since that time, zombie ideas are resilient and continue to stalk the face of the earth in original and mutated strains. The literature cited in the briefs is taken largely from the popular periodical literature of the time or government and Congressional publications that conscientious scholars of the history of economics really need to be familiar with. Such stuff is not yet quite so neatly sorted and indexed for our purposes as to facilitate entry into flow of actual policy debates outside the academic realm. The collection of Harvard student debating briefs used here is really a treasure chest (Pandora’s box?) waiting to be opened, filled with good, bad, and ugly arguments regarding international commercial policy.

Also thanks to another of Trump’s policy initiatives, Economics in the Rear-view Mirror has provided transcriptions of analogous old debating briefs on the subject of immigration into the U.S.

The eight debate topics concerning international trade policy were:

Resolved, That the time has now come when the policy of protection should be abandoned by the United States.

Resolved, That a high protective tariff raises wages.

Resolved, That it would be to the advantage of the United States to establish complete commercial reciprocity between the United States and Canada.

Resolved, That foreign-built ships should be admitted to American registry free of duty.

Resolved, That the United States should establish a system of shipping subsidies.

Resolved, That sugar should be admitted free of duty.

Resolved, That a system of sugar bounties is contrary to good public policy.

Resolved, That a system of duties on wool and woollens is undesirable.

 

_________________________________

Briefs for Debate on Current Political, Economic, and Social Topics.

Edited by
W. Du Bois Brookings, A.B. of the Harvard Law School
And
Ralph Curtis Ringwalt, A.B.
Assistant in Rhetoric in Columbia University

With an introduction by Albert Bushnell Hart, Ph.D.
Professor of Harvard University.
(1908)

[From the Preface:]

“The basis of the work has been a collection of some two hundred briefs prepared during the past ten years [ca. 1886-96] by students in Harvard University, under the direction of instructors. Of these briefs the most useful and interesting have been selected; the material has been carefully worked over, and the bibliographies enlarged and verified….

…” the brief is a steady training in the most difficult part of reasoning; in putting together things that belong together; in discovering connections and relations; in subordinating the less important matters. The making of a brief is an intellectual exercise like the study of a disease by a physician, of a case by a lawyer, of a sermon by a minister, of a financial report by a president of a corporation. It is a bit of the practical work of life.

_________________________________

PROTECTION AND FREE TRADE.

Question: ‘Resolved, That the time has now come when the policy of protection should be abandoned by the United States.’

Brief for the Affirmative.

General references:

Frédéric Bastiat, Sophisms of the Protectionists; W. M. Grosvenor, Does Protection Protect?; Henry George, Protection or Free Trade; J. S. Mill, Principles of Political Economy, II., Bk. V., Chap. x., § 1; article on Protection in Tariff Reform Series, IV., No. 12, p. 2 (September 30, 1891); Lalor’s Cyclopædia, II., 289; Nation, XXVIII., 161 (March 6, 1879); XXIX., 338 (November 20, 1879); XXXIV., 288 (April 6, 1882) ; LXXVI., 118 (February 8, 1883); J. G. Carlisle in Congressional Record, 1891-1892, p. 6910 (July 29, 1892); D. A. Wells in Forum, XIV., 697 (February, 1893); F. A. Walker in Quarterly Journal of Economics, IV., 245 (April, 1890); Edward Atkinson in Popular Science Monthly, XXXVII., 433 (August, 1890); Senator Vest in North American Review, Vol. 155, p. 401 (October, 1892); Harper’s Weekly, XXXVIII., 819 (September 1, 1894).

  1. Protection is unsound in theory:

J. S. Mill, Principles of Political Economy, II., 532. — (a) It shuts out what is ours by nature: Sophisms of the Protectionists, pp. 73-80. — (b) It raises unnatural obstacles to intercourse: Sophisms of the Protectionists, pp. 84-85. — (c) It can only raise prices by diminishing the quantity of goods for sale: Sophisms of the Protectionists, pp. 7, 17. — (d) It endangers the interests it aims to promote: Nation, XXXVI., 118. — (e) It may transfer but not increase capital: Sophisms of the Protectionists, p. 93. — (f) The doctrine of protection for revenue is inconsistent: J. S. Mill, Principles of Political Economy, II., 538. — (g) It is anti-social: Sophisms of the Protectionists, pp. 15, 127; Nation, XXXVI., 118; XXXVIII., 161.

  1. Protection is unsound in general practice.

(a) It makes capital and labor less efficient: J. S. Mill, Principles of Political Economy, II., 532, 539. — (b) It hurts our carrying trade: Nation, XXXVI., 118. — (c) It closes against us many of the world’s best markets: J. S. Mill, Principles of Political Economy, II., 537; Nation, XXVIII., 161; XXXVI., 118.

  1. Protection is not beneficial to any class.

(a) It raises prices to consumers: Popular Science Monthly, XXXVII., 433. — (b) It does not raise the wages of laborers: Congressional Record, 1891-1892, pp. 6910-6917; Popular Science Monthly, XXXVII., 433. — (c) It hurts farmers: Nineteenth Century, XXXII., 733 (November, 1892). — (d) It hurts the community by shutting off foreign markets: North American Review, Vol. 155, p. 401. — (e) It increases the cost of materials. — (f) It does not help us against pauper labor: Popular Science Monthly, XXXVII., 433. — (g) It does not benefit the majority: Nation, LV., 299 (October 20, 1892). — (h) Infant industries are not permanently aided: Quarterly Journal of Economics, IV., 245.

  1. Protection tends to run to extremes.

(a) It perverts taxation from its proper uses: Forum, XIV., 51 (September, 1892). — (b) It creates dangerous precedents: Ibid. — (c) Industries seek permanent protection: Nation. LV., 252 (October 6, 1892). — (d) It creates monopolies.

Brief for the Negative.

General references:

S.N. Patten, The Economic Basis of Protection; H. M. Hoyt, Protection versus Free Trade; Congressional Record, 1889-1890, p. 4248 (May 7, 1890); 1891-1892, p. 6746 (July 26, 1892); J. G. Blaine in North American Review, Vol. 150, p. 27 (January, 1890); William McKinley in North American Review, Vol. 150, p. 740 (June, 1890); R. E. Thompson, Social Science and National Economy, pp. 243-278; Lalor’s Cyclopædia, III., 413; Van Buren Denslow, Principles of Economic Philosophy, Chaps. xiii., xiv., xv., xvi.

  1. The policy of protection is sound in principle.

(a) It enables a country to fix the terms of exchange in foreign trade. — (1) Foreign demand for our commodities is necessarily great. — (2) Protection lessens our demand for foreign commodities. — (b) Protection is the best means of increasing the consumer’s rent.

  1. The policy of protection has proved beneficial in practice.

(a) Without it no country has secured a symmetrical development of its industries: Social Science and National Economy, p. 267. — (b) Every period of protection in the United States has been followed by great material prosperity.

  1. Protection secures a home market for commodities incapable of transportation abroad:

E.E. Hale, Tom Torrey’s Tariff Talks. — (a) It enhances values, especially the value of land: J. R. Dodge, How Protection Protects the Farmer.

  1. A protective tariff does not raise prices.

(a) The establishment of a new industry has invariably been followed by lower prices: Congressional Record, 1889-1890, p. 4248.—. (1) Steel rails.—(2) Glass and earthen ware.—(3) Wool.— (4) Tin-plate.

_________________________________

THE TARIFF AND WAGES.

Question:Resolved, That a high protective tariff raises wages.’

Brief for the Affirmative.

General references:

S. N. Patten, The Economic Basis of Protection, pp. 54-80; Lee Meriwether, ‘How Workingmen Live in Europe and America,’ in Harper’s Magazine, LXXIV., 780 (April, 1887); R. P. Porter, Bread Winners Abroad (People’s Library), Chaps. xvi., xxviii., xlix., li., liii., lvi., lxvi., lxvii., lxxxiv., civ.; Van Buren Denslow, Principles of Economic Philosophy, pp. 623-627.

  1. A high protective tariff raises wages theoretically.

(a) It causes more employers to compete for the hire of labor.—(1) By increasing the number of occupations and enterprises that can be carried on: R. E. Thompson, Social Science and National Economy, p. 248; Principles of Economic Philosophy, pp. 623-624. (b) It increases the amount of money available for the compensation of labor.—(1) By increasing the profits of manufacturers: Principles of Economic Philosophy, pp. 626-627. (c) It enables laborers to share in the natural resources of the country.—(1) By preventing competition with cheap foreign labor: The Economic Basis of Protection, pp. 64-70.

  1. A high protective tariff raises wages practically.

(a) In the United States, which furnishes the best example of a protective tariff, money wages are higher than in Europe.— (1) This is shown by the opinions of writers: Principles of Economic Philosophy, p. 527; Bread Winners Abroad; Consular Reports of the United States, No. 40, p. 304 (April, 1884). —(2) It is shown by the opinions of manufacturers: John Roach in International Review, XIII., 455 (November, 1882); J. M. Swank, Our Bessemer Steel Industry, p. 23; letters from the National Association of Wool Manufacturers and the Titus Sheard Co. in Congressional Record, 1891-1892, p. 6751 (July 26, 1892). (b) Wages have risen in other countries under a protective system. — (1) In Germany: Principles of Economic Philosophy, pp. 523-524; Consular Reports of the United States, No. 42, pp. 12, 13, 15 (June, 1884).—(2) In Canada: Principles of Economic Philosophy, pp. 666-668. (c) Real wages are higher in the United States than in Europe.—(1) An American workman can save more than a European: Consular Reports of the United States, No. 40, p. 304.—(2) His standard of living is higher: Harper’s Magazine, LXXIV., 780.

Brief for the Negative.

General references:

F. W. Taussig in Forum, VI., 167 (October, 1888); W. G. Sumner in North American Review, Vol. 136, p. 270 (March, 1883); J. Schoenhof, The Economy of High Wages, pp. 175-193; J. Schoenhof, Wages and Trade; ‘Labor, Wages, and Tariff,’ Tariff Reform Series, II., No. 21 (January 15, 1890); ‘Labor and the Tariff,’ Tariff Reform Series, I., No. 12, p. 2 (October 10, 1888).

  1. Arguments based on comparisons of wages in different countries are untrustworthy.

(a) Such comparisons prove too much: D. A. Wells, Practical Economics, p. 137. — (b) There is no uniform rate in any country. — (c) There are many local causes which must necessarily make wages higher in one country than in another. — (1) Natural advantages: D. A. Wells, The Relation of the Tariff to Wages, p. 2. — (2) Standing army service: Ibid. — (3) The question of unoccupied land: North American Review, Vol. 136, p. 270.

  1. Careful use of statistics shows that wages are relatively higher under a low tariff.

(a) The high rate of wages in the United States is determined by unprotected industries.— (1) There are more laborers connected with unprotected than with protected industries: J. L. Laughlin’s edition of J. S. Mill, Principles of Political Economy, p. 619. — (b) Wages in certain protected industries in the United States are lower than wages in the same industries in England. — (c) In protected industries in which wages are higher than abroad, they were higher before the existence of a protective tariff: Nation, XLVII., 327 (October 25, 1888). — (d) New South Wales is more prosperous than Victoria: Fortnightly Review, XXXVII., 369 (March, 1882).

  1. A protective tariff lowers wages by diminishing the amount of capital to be distributed for wages.

(a) The general productiveness of industry is less: Practical Economics, p. 135.— — (1) The effect of limiting the sale of commodities to a domestic market is evil: Practical Economics, p. 139. — (b) The proportion in which that produced is divided is less favorable to labor.—(1) The producer requires the same ratio of profit, while the number of laborers among whom the smaller wage-fund is divided is as large as before: North American Review, Vol. 136, p. 270.

  1. Real wages are less.

(a) The tariff increases the price of commodities and puts them out of the reach of the poorer classes: North American Review, Vol. 136, p. 270.

_________________________________

RECIPROCITY WITH CANADA.

Question: ‘Resolved, That it would be to the advantage of the United States to establish complete commercial reciprocity between the United States and Canada.’

Brief for the Affirmative.

General references:

Goldwin Smith, Canada and the Canadian Question, pp. 281-301; Handbook of Commercial Union (Toronto, 1888); Century, XVI., 236 (June, 1889); Forum, VI., 241 (November, 1888) ; VII., 361 (June, 1889); New Englander, LIII., 1 (July, 1890); North American Review, Vol. 148, p. 54 (January, 1889); Vol. 151, p. 212 (August, 1890); Vol. 139, p. 42 (July, 1884); Harper’s Magazine, LXXVIII., 520 (March, 1889).

  1. Greater freedom of trade between the United States and Canada is desirable.

(a) It would furnish the United States with much needed raw materials: Century, XVI., 236. — (1) Coal, iron, and other mineral products are extensive and easily accessible to the northern and middle states: Handbook of Commercial Union, pp. 72-85; North American Review, Vol. 139, p. 42. — (2) Agricultural products. — (b) It would open to us a large and convenient market for our manufactures: Handbook of Commercial Union, p. 249. — (c) Closer commercial relations would remove much of the present ill feeling, and international disputes would be avoided.

  1. Reciprocity would be advantageous economically.

(a) It would open up a great field for the investment of American capital: Handbook of Commercial Union, p. 247. — (b) It would do away with the enormous expense of maintaining an unnatural customs line four thousand miles long. — (c) By the settlement of the fishery question it would give our fishermen valuable privileges.

  1. Reciprocity is practical:

Handbook of Commercial Union, p. 111. — (a) Great Britain would not raise serious objections: Handbook of Commercial Union, p. 101 .— (1) English investments in Canada would be benefited by commercial prosperity. — (2) Greater commercial activity would establish confederation on a firm basis and give assurance that Canada would remain a part of the British domain. — (b) The loyalty of Canadians would not be affected. — (1) The common tariff would not discriminate against England. — (c) A common tariff could be agreed upon. — (1) The present policy of the United States is toward a reduction of tariffs, while that of Canada is toward an increase. — (2) Canada would be willing to make concessions, such as the adjustment of internal revenue. — (d) The reciprocity treaty of 1854 was a commercial success. — (1) Trade rose from seven millions to twenty: Encyclopedia Britannica, IV., 766. — (2) The abrogation of the treaty was due to national animosity caused by acts of the English during the civil war.

 

Brief for the Negative.

General references:

James Douglas, Canadian Independence, Annexation, and British Imperial Federation; Forum, VI., 451 (January, 1889); J. N. Larned, Report to the Secretary of the Treasury on the State of Trade Between the United States and British Possessions in North America, January 28, 1871; Penn Monthly, V., 529 (July, 1874); Congressional Globe, 1864-1865, pp. 229-233 (January 12, 1865).

  1. Complete commercial reciprocity is impracticable.

(a) The commercial policies of Great Britain and the United States are conflicting. — (b) A common tariff could not be decided upon without detriment to one country. — (c) Internal revenue stands in the way.—(1) Excise taxes and internal revenue would have to be made equal; but excise is necessary to Canada, while it is not unlikely that we shall do away with our internal revenue: Forum, VI., 451.

  1. Complete reciprocity would be contrary to good public policy.

(a) It would result in loss of revenue. — (b) In case of war with Great Britain the frontier would be in a bad condition, and our whole tariff system would be torn asunder.

  1. Complete reciprocity would be economically disastrous.

(a) American and Canadian products are not supplementary, but competitory. — (b) Cheaper wages and cheaper raw material would be an inducement for our capital to move to Canada, and would also lower wages in the United States. — (c) We should lose much through emigration to Canada. — (d) It would give Canada the benefit of the market which we hav

e built up for ourselves by protection: Penn Monthly, V., 531.

  1. Historically, reciprocity with Canada has proved injurious.

(a) The United States tried commercial reciprocity with Canada in 1854, but abrogated the treaty in 1866.

_________________________________

FREE SHIPS.

Question: ‘Resolved, That foreign-built ships should be admitted to American registry free of duty.’

Brief for the Affirmative.

General references:

D. A. Wells, The Decay of Our Ocean Mercantile Marine; John Codman, Free Ships; J.D.J. Kelly, The Question of Ships; North American Review, Vol. 142, p. 478 (May, 1886); House Reports, 1889-1890, No. 1210, Minority Report; 1882-1883, No. 1827, Views of the Minority; 1891-1892, No. 966; 1887-1888, No. 1874; Congressional Record, 1890-1891, p. 1044 (January 8, 1891); Congressional Globe, 1871-1872, Part 3, p. 2241 (April 6, 1872).

  1. A change in our navigation laws is necessary.

(a) Under their restrictions American shipping has suffered. — (1) Through heavy duties on ships. — (b) Though heavily protected, the ship-building industry has not thrived. — (1) The cost of labor is too great. — (c) American capital has been forced abroad. — (d) The present provision for the limited admission of foreign ships is inadequate. — (e) The development of inventive genius is prevented.

  1. Free ships furnish the only practicable remedy:

The Question of Ships, Chap. v. — (a) They enable Americans to compete on equal terms for world’s commerce. — (1) Ships can be bought at the lowest price. — (b) Carrying trade should not be sacrificed to ship-building.—(1) It employs fifty times as many men: The Question of Ships, p. 31. — (c) American ship-building would not be seriously affected.— (1) Only iron ships are concerned. — (d) The success of the plan is well illustrated by Germany’s policy.

  1. Subsidizing schemes are impracticable and inefficient:

The Question of Ships, Chap. iv. — (a) Subsidies large enough to be efficient would be too great a tax on the people. — (1) The cost of building ships is one-third greater than in England: John Codman, Free Ships. — (b) They must be permanent. — (c) They have already been unsuccessfully tried in the United States. — (d) They have failed in France. — (1) Ship-building has not been built up in ten years’ trial. — (e) England’s supremacy is not due to subsidizing: The Decay of Our Ocean Mercantile Marine, pp. 29-45. — (1) No payments are made to sailing vessels. — (2) Compensation is given only for carrying mails, and for building according to admiralty requirements.

Brief for the Negative.

General references:

W. W. Bates, American Marine; C. S. Hill, History of American Shipping; H. Hall, American Navigation; North American Review, Vol. 148, p. 687 (June, 1889); Vol. 154, p. 76 (January, 1892); Vol. 158, p. 433 (April, 1894); House Reports, 1891-1892, No. 966, Views of the Minority; 1887-1888, No. 1874, Views of the Minority, p. 10; 1882-1883, No. 1827; 1869-1870, No. 28; Nelson Dingley, Jr., in Congressional Record, 1890-1891, p. 997 (January 7, 1891).

  1. The lack of free registry was not responsible for the decline in American shipping.

(a) Under the present laws our merchant marine reached its height. — (b) The decline was due to other causes. — (1) To the destruction of commerce by English-built cruisers: American Marine, Chap. ix. — (2) To the commercial depression following war. — (3) To mechanical changes. — (x) From wood to iron. — (y) From sail to steam.

  1. Free registry offers no material advantages.

(a) American capital now invests in foreign-built ships. — (1) ‘Whitewashed’ sales: American Navigation, p. 75. — (b) The advantage of flying American flag would be subject to abuse.

  1. Free registry involves grave evils.

(a) Economic. — (1) It would annihilate ship-building in the United States. — (2) It would withdraw millions of capital from the country. — (b) National. — (1) It would cripple us in time of war. — (x) We should have no trained workmen. — (y) We should have no shipyards to build in an emergency.

  1. There are better alternatives than free registry.

(a) The removal of duties on materials. — (b) Sufficient mail subsidies to American-built ships: American Navigation, p. 77. — (c) A change in taxation from the principal invested in ships to net profits.

_________________________________

SHIPPING SUBSIDIES.

Question: ‘Resolved, That the United States should establish a system of shipping subsidies.’

Brief for the Affirmative.

General references:

W. W. Bates, American Marine; House Reports, 1889-1890, No. 1210; C. S. Hill, History of American Shipping; House Reports, 1888-1889, No. 4162, Views of the Minority, p. 5; Congressional Record, 1890-1891, p. 997 (January 7, 1891), p. 3355 (February 26, 1891); Statement of Captain W. W. Bates in House Reports, 1889-1890, No. 1210, p. 220; Overland Monthly, I., 462 (May, 1883); H. Hall, American Navigation.

  1. The merchant marine of the United States is at present in a deplorable condition and ought to be built up:

House Reports, 1889-1890, No. 1210, pp. i-vi. — (a) A national marine is of the greatest importance to the wealth and the commercial prosperity of a nation: Lalor’s Cyclopædia, II., 987; J.D.J. Kelly, The Question of Ships, p. 108. — (1) It is essential to naval power. — (2) To the development of resources. — (3) To national unity and individualism. — (b) The United States has the necessary qualifications for the marine industry: The Question of Ships, Chap. i.; American Navigation, Chap. ii. — (1) In 1856 the United States merchant marine was the most extensive in the world. — (2) Our extensive sea-coast naturally fosters a maritime spirit. — (3) We have abundant natural resources. — (4) Extensive commerce. — (5) Great ship-building interests.

  1. The subsidy system is a desirable means of building up the marine.

(a) It is preferable to the policy of free ships. — (1) Such a policy would destroy our ship-building industry: American Navigation, Chap. vii. — (b) Subsidies given to vessels for mail service would greatly encourage commerce. — (1) By insuring regular service: American Navigation, p. 77; Congressional Record, 1885-1886, p. 4009 (April 30, 1886). — (c) Vessels subsidized could be put under contract to serve the United States in case of war: American Navigation, pp. 83-86. — (d) It is an economical system. — (1) The total payments would not exceed $5,000,000 per annum. — (2) The earnings of the foreign mail service, which amount to $10,000,000 per annum, could fittingly be used for subsidies: Congressional Record, 1889-1890, p. 6996 (July 7, 1890).

  1. Subsidies are necessary.

(a) The cost of American ships and their running expenses are greater than those of foreign vessels. — (b) The high subsidies given to foreign lines make it impossible for American lines to compete without like subsidies.

  1. Subsidies have proved successful in practice:

American Marine, pp. 325-327. — (a) We have tried such a system and found it effective: W. S. Lindsay, Merchant Shipping, IV., 194-228. — (b) Nearly all foreign nations maintain shipping subsidies: Congressional Record, 1890-1891, pp. 3359-3362 (February 26, 1891). — (c) They have been successful in France: House Reports, 1889-1890, No. 1210, pp. ix-xv. — (d) Great Britain, the foremost maritime country, has steadily adhered to a system of bounties: Congressional Record, 1890-1891, pp. 1001-1003 (January 7, 1891).

Brief for the Negative.

General references:

House Reports, 1889-1890, No. 1210, Minority Report, p. xxxix.; D.A. Wells, Our Merchant Marine; D.A. Wells, The Decay of Our Ocean Mercantile Marine; John Codman, Free Ships; John Codman, Shipping Subsidies and Bounties; Congressional Record, 1890-1891, pp. 3348, 3368, 3383 (February 26, 1891); 1889-1890, p. 6959 (July 3, 1890); House Reports, 1888-1889, No. 4162; J. D. J. Kelly, The Question of Ships.

  1. Subsidies are politically objectionable.

(a) They have proved and always will prove inducements to corrupt legislation. — (b) They create and foster a privileged class at the expense of the whole people: Our Merchant Marine, p. 141; Free Ships, p. 15. — (c) The practice would establish a bad precedent: House Reports, 1889-1890, No. 1210, pp. xl., xlii.

  1. Subsidies are economically objectionable:

Congressional Record, 1890-1891, p. 3352. — (a) They are merely temporizing measures: The Decay of Our Ocean Mercantile Marine, p. 25. — (b) They would be a tremendous cost: House Reports, 1888-1889, No. 4162, p. 4. — (c) They would not contribute to the general prosperity of the country: House Reports, 1888-1889, No. 4162, pp. 2-3. — (1) They would not benefit commerce. — (x) Foreign vessels now carry as cheaply as it can be done. — (2) They would benefit one industry at the expense of others. — (3) As profit would come wholly from subsidies, shippers would become uneconomical and the advantages of competition would be lost.

  1. There is no truth in the statement that shipping subsidies have built up merchant marines.

(a) Great Britain does not subsidize her vessels: The Decay of Our Ocean Mercantile Marine, p. 29; House Reports, 1889-1890, No. 1210, pp. xlii., 1. — (1) British mail subsidies are for actual service rendered as shown by the exacting rules and penalties for non-performance of contracts. — (b) The French system has not been successful: House Reports, 1888-1889, No. 4162, p. 3; 1889-1890, No. 1210, pp. 1-lx. — (c) Our own experience has been unfavorable. — (1) The Collins line in 1847: Congressional Record, 1890-1891, p. 3386.

  1. The best remedy for American shipping is free ships:

Our Merchant Marine, pp. 95-128; North American Review, Vol. 142, pp. 481-484 (May, 1886). — (a) Free ships would at least allow Americans to compete on equal terms for the commerce of the world.

 

_________________________________

FREE SUGAR.

Question: ‘Resolved, That sugar should be admitted free of duty.’

Brief for the Affirmative.

General references:

‘Sugar and the Tariff,’ Tariff Reform Series, III., No. 12, p. 174 (July 30, 1890); Harper’s Weekly, XXXVIII., 602 (June 30, 1894), 771 (August 18, 1894), 819 (September 1, 1894); Nation, LIX., 74 (August 2, 1894), 112 (August 16, 1894); Congressional Record, 1889-1890, p. 10,631 (September 27, 1890).

  1. The question of protection does not enter.

(a) We produce only ten per cent, of the sugar we use: Princeton Review, VI., 322 (November, 1880). (b) The established industry can be more economically protected by bounties.

  1. The tariff is a burden on the poor.

(a) The poor man must pay more in proportion to his ability than the rich: C. D. Wright in Seventeenth Annual Report of Massachusetts Bureau of Statistics of Labor, p. 266; W. O. Atwater in American Public Health Association, XV., 208. — (1) Carbohydrates are necessary to life. — (2) Sugar is the most economical carbohydrate. — (3) The laboring man consumes the greatest proportion of this constituent: American Public Health Association, XV., 216.

  1. The sugar tariff is a check to the country’s development.

(a) It discourages industries in which sugar is a raw material. — (1) The preserving industry. — (2) The condensed milk industry. — (3) The refining industry. — (b) It injures foreign commerce. — (1) With Brazil and Cuba. — (2) Germany has retaliated for our tariff by putting a tax on American beef: Harper’s Weekly, XXXVIII., 1058 (November 10, 1894).

  1. Sugar taxes are a great source of corruption.

(a) They enable importers to defraud the government by manipulating the grades of sugar. — (b) They give rise to political corruption such as has disgraced the Senate. — (1) By fostering the sugar trust: Nation, LVIII., 440 (June 14, 1894); LIX., 71, 93, 112; Harper’s Weekly, XXXVIII., 602, 771, 819; Tariff Reform Series, VII., No. 2, p. 28 (July 1, 1894).

  1. The sugar tax is not necessary for revenue.

(a) If the revenues fall short, the deficiency can be made up better by replacing the higher taxes on malt liquors and tobacco.

Brief for the Negative.

General references:

Congressional Record, 1893-1894, Appendix, p. 1178 (August 13, 1894), p. 634 (January 23, 1894); 1889-1890, Appendix, p. 437 (May 20, 1890); Harper’s Weekly, XXXVIII., 218 (March 10, 1894); Tariff Hearings Before the Committee on Ways and Means, 1893, pp. 505, 520, 542.

  1. A tax on sugar is a just way of raising revenue:

Congressional Record, 1893-1894, Appendix, p. 1182. — (a) It is evenly distributed: Ibid. — (1) It reaches consumers in proportion to their incomes. — (2) Sugar is to a great extent an article of voluntary consumption.

  1. It is a desirable way of raising revenue.

(a) It is the only tax which furnishes a steady, reliable revenue, capable of computation beforehand. — (b) It is an easy tax to collect. — (c) Precedent has established sugar as a fitting article for taxation: D. A. Wells in Princeton Review, VI., 323 (November, 1880); Congressional Record, 1893-1894, Appendix, pp. 1180-1186. — (1) It has heretofore furnished one-fourth of the total revenue: D. A. Wells, The Sugar Industry of the United States and the Tariff, p. 9.

  1. The tax is necessary to encourage the American sugar industry:

Congressional Record, 1893-1894, Appendix, p. 632. — (a) The beet and sugar industries are difficult to establish. — (1) They require a large outlay of capital at the beginning. — (2) The return on the investment is small. — (3) The industries are still experimental. — (b) American producers require a special protective tax to offset the large bounties which foreign countries pay to their producers.

  1. The objections to the tax are unsound.

(a) The sugar-refining trust would remain even if sugar were admitted free. — (1) As nearly all of the sugar admitted to the United States is raw, it would still have to pass through the refineries. — (b) The frauds against the government, due to the manipulation of grades, are not an inherent result of the tax.

_________________________________

SUGAR BOUNTIES.

Question: ‘Resolved, That a system of sugar bounties is contrary to good public policy.’

Brief for the Affirmative.

General references:

D. A. Wells, Recent Economic Changes, pp. 295-309; Lalor’s Cyclopædia, II., 99; Fortnightly Review, XLII, 638 (November, 1884) ; Nation, XLV., 164 (September 1, 1887); XLII, 420 (May 20, 1886); Congressional Record, 1889-1890, pp. 10,712-10,716 (September 30, 1890), Appendix, p. 391.

  1. The bounty system is unconstitutional.

(a) It is legislation in favor of a class: Nation, XLVII., 24 (July 12, 1888); Congressional Record, 1889-1890, pp. 10,712-10,716, Appendix, p. 391; Loan Association v. Topeka, 120 Wallace, 663-664.

  1. The bounty system is burdensome on the people:

Nation, XLIV., 484 (June 9, 1887). — (a) The people are compelled to pay the bounty: Fortnightly Review, XLII., 638. — (b) They are compelled to pay the highest cost of production for sugar: Fortnightly Review, XLII., 638. — (c) They are compelled to pay for the expensive system of administration.

  1. The bounty system gives rise to fraud.

(a) It places a great amount of money and patronage in the hands of political parties: Congressional Record, 1889-1890, Appendix, p. 391. — (b) The intricate system of bounty payments enables producers to defraud the government: Recent Economic Changes, pp. 295-298.

  1. The bounty system is injurious to commerce.

(a) It deranges prices. — (1) The producer is led to disregard the law of supply and demand: Fortnightly Review, XLII., 638. — (b) It makes foreign exchange uncertain: Nation, XLV., 164. — (1) By causing alternate over-production and under-production: Recent Economic Changes, pp. 295-309. — (c) It enables producers to control the markets.

  1. The bounty system is unnecessary for the development of the industry.

(a) The United States has as good facilities for raising beets as any other country. — (b) The sugar industry is not an infant industry.

  1. The bounty system has proved a failure in Europe:

Nation, XLVI., 45 (January 19, 1888); Recent Economic Changes, pp. 295-309; Lalor’s Cyclopædia, II., 99. — (a) The beet-sugar industry was fostered at the expense of cane sugar: Nation, XLV., 164. — (b) International complications arose: Saturday Review, LXIV., 142 (July 30, 1887), 847 (December 24, 1887).

Brief for the Negative.

General references:

Essay on ‘Industry and Commerce’ in Works of Alexander Hamilton, III., 366; Congressional Record, 1889-1890, p. 4266 (May 7, 1890); Senators Allison and Sherman in Congressional Record, 1888-1889, pp. 888-895 (January 17, 1889).

  1. The sugar industry is highly desirable.

(a) The importance of sugar as a food is constantly increasing: Congressional Record, 1889-1890, p. 4266. — (b) The industry will be national, not sectional: Congressional Record, 1888-1889, p. 892; 1889-1890, p. 4515 (May 10, 1890). — (c) Beets do not exhaust the soil: Congressional Record, 1889-1890, p. 4266.

  1. The sugar industry would bring general economic advantages.

(a) It would keep at home money now sent abroad in payment for sugar. — (b) Capital greatly exceeding the amount of the bounty would be invested in the industry. — (c) The industry would create a new and a large demand for labor, both agricultural and mechanical.

  1. The bounty system is the best means of establishing the sugar industry.

(a) Protective duties are inadequate. — (1) Bounties paid by foreign countries tend to counteract our tariff. — (2) In the past import duties have failed. — (b) Bounties are necessary to tide the industry over the critical time of beginning: Congressional Record, 1889-1890, p. 4515. — (1) Establishment is difficult and expensive. — (2) There is small inducement for capital. — (3) Beet and sorghum sugar industries are more or less experimental. — (c) Bounties have been successful in establishing industries abroad. — (1) Beet-sugar industry in Germany: Congressional Record, 1889-1890, pp. 4266, 4431 (May 9, 1890).

  1. The bounty system is constitutional.

(a) The bounty is extended to anyone who is willing to undertake the production of sugar: American Law Register and Review, XXXI., 289 (May, 1892).

_________________________________

DUTIES ON WOOL AND WOOLLENS.

Question: ‘Resolved, That a system of duties on wool and woollens is undesirable.’

Brief for the Affirmative.

General references:

F. W. Taussig in Quarterly Journal of Economics, VIII., 1 (October, 1893); North American Review, Vol. 154, p. 133 (February, 1892); ‘Wool and Tariff,’ Tariff Reform Series, III., No. 19, p. 342 (November 15, 1890); ‘The Wool Question,’ Tariff Reform Series (Report of Ways and Means Committee on the Springer Bill), V., No. 1, p. 1 (March 15, 1892).

  1. Duties on wool and woollens have failed to bring beneficient results.

(a) Wool-growing has not prospered. — (1) The United States cannot raise grades of wool that will compare in quality with the better grades of foreign countries. — (x) Owing to climate: Quarterly Journal of Economics, VIII., 18. — (b) Woollen manufacturers produce only the cheapest grades of woollens. — (c) Under the tariff American producers have succeeded in producing but a small quantity of woollens in comparison with foreign importations: Quarterly Journal of Economics, VIII., 28-29; Tariff Reform Series, III., No. 19, p. 359.

  1. The removal of duties on wool does not hurt woolgrowers.

(a) The grades of wool raised by American growers are not subject to foreign competition. — (1) In these grades the American producer has an equal advantage with foreign producers: Quarterly Journal of Economics, VIII., 5-20.

  1. Free woollens are not injurious to manufacturers.

(a) They do not injure the production of cheap grades of woollens for the American market. — (1) The American manufacturer, owing to the greater efficiency of his machinery and the small necessity for hand labor, can compete on equal terms in these grades.

  1. The removal of duties on wool is a benefit to manufacturers.

(a) It enables them to engage in the manufacture of finer grades of woollens: Quarterly Journal of Economics, VIII., 32-33. — (1) By giving them free raw material of finer grades. — (b) It gives them a larger assortment of wools from which to select their grades: Congressional Record, 1887-1888, pp. 6519-6530 (July 19, 1888). (c) It enlarges their trade with South America: Nation, XLVI., 500 (June 21, 1888).

  1. Duties are unjust to consumers.

(a) They require them to pay a high price for woolens which are not made in America. — (1) This is shown by the constant increase in the importations of the finer grades of woollens in spite of the high tariff.

Brief for the Negative.

General references:

Bulletin of National Association of Wool Manufacturers, XVIII., 1888, Nos. 2, 3; XXII., 268 (September, 1892); XXIII., 275 (December, 1893); XXII., 1 (March, 1892); XXL, 333 (December, 1891); XXII., 115 (June, 1892); W. D. Lewis, Our Sheep and the Tariff (Publications of the University of Pennsylvania), Chaps. i., vii.; Congressional Record, 1893-1894, Appendix, pp. 1064, 1172.

  1. Duties on wool are necessary to protect the sheep-raising industry:

Our Sheep and the Tariff, Chap. vii. — (a) Foreign competition is especially active in this industry. — (1) Australia and the Argentine Republic have superior natural advantages.

  1. Duties on woollens are necessary to protect manufacturers:

Bulletin of National Association of Wool Manufacturers, XXII., 133. — (a) Foreign manufacturers have an advantage in cheap labor. (b) Foreign manufacturers have as good machinery as manufacturers in the United States. — (1) American machinery is used extensively abroad. — (c) The return on investments in the United States is less than it is abroad. — (1) A larger capital is required to produce an equivalent amount of woollens: Bulletin of National Association of Wool Manufacturers, XXII., 136.

  1. The history of the United States shows that duties have been successful in building up the wool and woollen industries:

Bulletin of National Association of Wool Manufacturers, XVIII., 234. — (a) The production of wool has greatly increased since the system was begun. — (b) The woollen industry is four times as large as in 1860: Bulletin of National Association of Wool Manufacturers, XXII., 3. — (c) Under periods of high protection the industries have been most prosperous.

  1. The duties have benefited the consumers:

Bulletin of National Association of Wool Manufacturers, XXII., 119. (a) They have reduced the price of woollens to less than half what it was thirty years ago. — (1) By causing active competition and rapid improvements in machinery: Bulletin of National Association of Wool Manufacturers, XXII., 119.

 

Source: W. Du Bois Brookings and Ralph Curtis Ringwalt, eds., Briefs for Debate on Current Political, Economic, and Social Topics. New York: Longmans, Green, and Co., 1908, pp. 96-117.

Image Source:  Cartoon by John S. Pughe published in Puck , September 15, 1897. Library of Congress Prints and Photographs Division Washington, D.C. 20540.

Categories
Columbia Curriculum Germany

Columbia. Political Economy Courses Compared to Courses at the University of Berlin, 1897

___________________

 

An excerpt from a newspaper report comparing political economy as taught in New York at Columbia University with political economy as taught in Berlin was published in the Columbia University Bulletin in 1897.  The unnamed author of the report concluded that “the primacy which Germany enjoyed a few years ago has passed away”. Compare this to a report (1884) overflowing with praise for the research “seminary” of  German universities.

___________________

In the Evening Post of October 25, 1897, will be found an interesting discussion of the value of German university degrees in comparison with similar honors in American universities. The writer, who is apparently a student in the University of Berlin, holds that the requirements for the degree of Ph. D. are higher in several American institutions than in the average German university. His points are, first, that it takes a shorter time to obtain the degree in Germany than from any of the reputable American universities; and second, that the average size and value of the dissertations of Harvard and Columbia doctors of philosophy are certainly greater than those of the German universities, with the exception, probably, of Berlin. Indeed, he concludes, “the progress of American universities has been so rapid in recent years, and the entrance requirements have been so largely increased, that the bachelor’s degree is actually approaching the German doctorate in essential worth.” A few selections from the body of the article, comparing the instruction in political science at Columbia with that given at Berlin, are of special interest.

“Further light on the question will be thrown by a comparison of the courses of lectures in American and German universities. Confining attention to the various studies in the domain of political economy and social science, we may select Berlin as the strongest representative of German Institutions.* * * * Of the American schools of political science, it is not easy to select the strongest. Columbia is usually regarded as the best equipped, although several others are but little inferior. Let us compare, then, the courses offered at Columbia and Berlin in political economy.

“At Berlin, Professor Wagner gives three courses, aggregating ten hours, that cover the field of general and theoretical economics, and practical economics, including money and banking, etc. At Columbia, almost precisely the same field is covered by Professor Mayo-Smith’s “Historical and Practical Economy,” running through three semesters and aggregating nine hours. Almost the only difference is that Professor Wagner devotes more time to agricultural economics, a subject that has as yet received little attention in American schools of political economy. In finance Professor Wagner offers a four-hour course for one semester. Professor Seligman at Columbia covers the same ground, with more discrimination, in a two-hour course running two semesters. He also offers in alternate years a two-hour course on the financial history of the United States.

“In economic or industrial history Columbia stands the comparison very well. It has an introductory course on the economic history of Europe and America conducted by Professor Seligman and Mr. Day, and an advanced course on the industrial and tariff history of the United States by Professor Seligman. The two courses aggregate the same number of hours as Professor Schmoller’s “practical political economy,” which is nothing but industrial history, and history of Prussia at that—a course valuable to the specialist, but not of great value to the average American student. Professor Meitzen also gives a course on the history of agriculture, but it concerns the early land systems of Europe and other subjects that can have no application to American conditions. The essential forms of land tenure are described at Columbia in Professor Mayo-Smith’s historical political economy.

“In the field of statistics, the subject of demography or population statistics is treated at Berlin by Professor Boeckh in a two-hour course, and at Columbia by Professor Mayo-Smith in a similar course. Economic statistics are treated by Professors Meitzen and Mayo-Smith in much the same manner, while the history, theory, and technique of statistics receives attention in both institutions.

“At Berlin, Professor Wagner reads a critique of socialism and Dr. Oldenburg gives its history. The two courses aggregate the same number of hours as Professor Clark’s course on socialism at Columbia. Professor Clark’s criticism of “scientific socialism” is at least equal to that of any German professor, and it proceeds from the Anglo-Saxon point of view. In a second semester Professor Clark deals with projects of social reform, especially those of American origin. Somewhat similar is Dr. Oldenburg’s course on Socialpolitik at Berlin, and Dr. Jastrow reads in addition a course on labor legislation.

“In social science Columbia is clearly in advance of Berlin. Sociology is scarcely recognized at the German universities, but at Berlin Dr. Simmel, privat-docent, offers a two-hour course on sociology and political psychology. This is the nearest approach to a study of the growth and structure of society that one finds at Berlin. Columbia, on the other hand, offers a course on the evolution of society and social institutions, with a review of the principal theoretical writers, and another course on sociological laws. These are both given by Professor Giddings, who also reads courses on crime and pauperism. No such practical study of these problems is made in Berlin.

“Several minor courses are offered at each university—as, for example, railway problems—and all of the professors conduct seminars for the purpose of encouraging and supervising original investigations. The only subject in which Berlin offers superior advantages is agricultural economics, while Columbia is doing much more work in both theoretical and practical social science. Two courses remain to be mentioned. One of these is a course by Dr. Jastrow at Berlin on the literature and methodology of all the political sciences, an introductory course of considerable value to freshmen, which has no parallel in any other German or American university known to the writer. But Columbia offers a course that can scarcely be duplicated in Germany, namely, the abstract theory of political economy given by Professor Clark, one of the acutest and most original thinkers of our day. It is a course that is taken by not more than a dozen or fifteen men, but they are advanced students who can appreciate such a course. Professor Clark’s power of inspiring young men to do theoretical work of high quality is evidenced by the writings of such men as the late Dr. Merriam, of Cornell, and Professor Carver, of Oberlin College. But in Germany pure theory has been neglected since the time of Hermann. Only now, as the result of an impulse proceeding from Austria, is theory regaining its place in German economic circles. Professor Dietzel and some of the other younger scholars are doing good work in this line, which is hardly comparable, however, with that of Professors Clark, Patten, etc., in the United States, and Marshall in England. German economists are making valuable contributions to economics in other ways, but the primacy which Germany enjoyed a few years ago has passed away.”

 

Source: Columbia University Bulletin, Vol. XVIII (December, 1897), pp. 67-69.

Image Source: The University of Berlin between ca. 1890 and ca. 1900. Library of Congress Prints and Photographs Division Washington, D.C. Digital ID: ppmsca 00342.

 

Categories
AEA Economists

AEA Twenty-fifth Anniversary Celebration, NYC 1909

MAYOR McCLELLAN, OF NEW YORK, ADDRESSING THE CONVENTION OF HISTORIANS AND ECONOMISTS AT CARNEGIE HALL, DECEMBER 27.

Others on the platform, beginning at left, are: William Jay Schieffelin, Isaac N. Seligman, Davis R. Dewey, John B. Clark, Albert Bushnell Hart, William M. Sloane, Ambassador James Bryce, Governor Hughes, Nicholas Murray Butler, Frank A. Vanderlip, Waldo Lincoln and Edwin R. A. Seligman.

____________________________________

The last posting came from the 25th anniversary celebration of the University of Chicago. It just so happens that I came across a clipping from the New York periodical The Independent in the John Bates Clark Papers in the Columbia University Archive that was about the joint 25th anniversary celebration of the American Historical Association and the American Economic Association held in New York City from December 27-31, 1909. Political Science, Sociology and Labor Legislation Associations also participated in the meetings. The report includes several photos of the men who were the movers-and-shakers of their respective associations (though none from the “playlet” and tableaux provided by the “ladies’ reception committee of the Waldor Astoria”).

As I like to provide the visitors of Economics in the Rear-View Mirror both accurate transcriptions and interesting images from yore, I hunted down scanned copies of The Independent at www.archive.org and www.hathitrust.org to extract a rough text file and better images than my amateur photographs of those in John Bates Clark’s clipping to create this posting.

 

____________________________________

A Notable Gathering of Scholars

BY REUBEN G. THWAITES

[Dr. Thwaites is secretary and superintendent of the State Historical Society of Wisconsin. He was for ten years managing editor of the Wisconsin State Journal and is well known as an author and editor of historical works. He attended the recent convention in New York as a delegate from Wisconsin. — EDITOR.]

 

THE twenty-fifth anniversary celebration of the American Historical Association and the American Economic Association, held in New York City from December 27 to 31, brought together the largest and doubtless the most distinguished assemblage of students of the social sciences ever convened in this country. In addition to the meetings of the two principal societies, which thus rounded out the quarter-centenary of their existence, were conferences by seven closely-related organizations—the American Political Science Association, the American Statistical Association, the American Sociological Society, the American Association for Labor Legislation, the American Social Science Association, the Bibliographical Society of America, and the American Society of Church History. Eleven hundred persons, engaged either in teaching or studying these several specialties, were gathered here from nearly every State or important institution of learning in the Union, and meetings, either — singly or jointly, occupied four busy days.

The attendance of several representative scholars from both Europe and Asia, who took part in many of the discussions, some of whom presented formal papers, and whose presence was recognized by numerous receptions and other social functions in their honor, gave to the gathering much of the significance of an international congress. Most prominent among the foreigners was the British Ambassador, Mr. Bryce, whose appearance at any of the meetings was invariably greeted with spontaneous applause, and whose many impromptu responses to calls by chairmen and toastmasters were never happier than on this anniversary occasion, into whose buoyant spirit he appeared keenly to enter.

Among other prominent foreign guests were: G. W. Prothero, of London, editor of the Quarterly Review, and former president of the Royal Historical Society; Prof. Herbert A. L. Fisher, fellow of New College, Oxford; Camille Enlart, director of comparative sculpture, of the Trocadero; Eduard Meyer, professor of ancient history, University of Berlin, exchange professor of Harvard; Dr. Cellenbrander, advisory secretary of the Dutch commission on governmental historical publications; Prof. Rafael Altamira y Crevea, professor-elect in the University of Madrid; Dr. Higgs, representing the Royal Economic Society of Great Britain, and Signor Maffeo Pantaleoni, of Rome, attending the Economic convention.

On occasions such as this presidential addresses are generally didactic, and by many of the older habitués are scrupulously avoided. But President Hart, of the Historical Association; President Lowell, of the Political Science, and President Dewey, of the Economic, always have something worth saying, and did not lack large and interested audiences. Dr. Hart’s discussion of “Imagination in history” was keen in its penetration and aglow with humor; he dwelt on the practical importance of the imaginative faculty on the part of the historian, but pointed out its manifest dangers, arising from a disposition to overemphasize dramatic episodes that really are rare in the history of a nation, whereas the most vital factors in its development are generally slow moving and commonplace. Dr. Lowell discussed “The physiology of politics”; while not deprecating the importance of library collections in the study of political science, the most useful laboratory work is, he said, the observation of the practical workings of political institutions, about which we are still insufficiently informed. Dr. Dewey spoke of “Observation in economics”; his thought being much in line with that of President Lowell, that field observation is of greater value than closet study, altho both are essential.

Ex-Presidents of the American Economic Association (S. N. Patten in the center, then clockwise from upper left are R. T. Ely, J. B. Clark, J. W. Jenks, F. W. Taussig.)
Ex-Presidents of the American Economic Association (S. N. Patten in the center, then clockwise from upper left are R. T. Ely, J. B. Clark, J. W. Jenks, F. W. Taussig.)

 

 

 

 

EARLY PRESIDENTS OF THE AMERICAN HISTORICAL ASSOCIATION Photograph by Brady, Washington, D. C. Beginning at left: William F. Poole, Justin Windsor, Charles Kendall Adams, George Bancroft, John Jay, Andrew D. White, Herbert B. Adams standing in rear.
EARLY PRESIDENTS OF THE AMERICAN HISTORICAL ASSOCIATION
Photograph by Brady, Washington, D. C.
Beginning at left: William F. Poole, Justin Windsor, Charles Kendall Adams, George Bancroft, John Jay, Andrew D. White, Herbert B. Adams standing in rear.
LIVING EX-PRESIDENTS OF THE AMERICAN HISTORICAL ASSOCIATION. (Clockwise from upper-left) James Ford Rhodes, Goldwin Smith, James Schouler, James Burrill Angell.

 

The programs of the nine associations were crowded with what the sporting gentry style “events,” but popular interest appeared chiefly to be with the joint sessions, some of which would have been noteworthy occurrences had they not jostled one another in this remarkable conference week. Wednesday morning’s joint session of the Historical and Economic associations called out a large and brilliant audience, with President Lowell in the chair. The general topic was “British constitutional and political development with special reference to the Gladstone centenary.” Professor Wrong, of Toronto, was hopeful concerning “Canadian nationalism and the imperial tie”; Professor Fisher, of Oxford, gave an exceptionally clear and informing account of the South African union, and Ambassador Bryce spoke forcefully on recent English history in its constitutional aspects.

Another excellent joint session was that held by the political scientists and the, association charging itself with scientific suggestion in the matter of labor legislation. The relation of the State to labor was interestingly and suggestively discussed by delegates from the Mississippi basin, where, perhaps, the best opportunities just now exist for trying out some of the theories of economic and sociological reformers.

The Historical Association, with its three thousand members, has of late years been doing its most effective work thru an admirable and impressive congerie of, commissions and conferences. This year’s meeting was chiefly noticeable for the variety and general success of these conferences, several of which were generally in session at one and the same time. One morning the topics were ancient, medieval, and American history and the treatment of archives. Later in the day the historians were conferring upon modern, European and American history, and relative to the methods and aims of State and local historical, a fertile theme, now engaging much attention in all parts of the country. One of the most interesting of the conferences was devoted to the consideration of “The contribution of the romance nations to the history of America,” in which Spain, France, Portugal and the Latin- American republics were represented either by scholars from those nations or by American specialists in the topics treated. A general session on Southern history brought out an interesting group of papers; while another on the work of historical societies in Europe was noticeable for careful reports from representatives of Great Britain, Germany, France. Holland and Spain, by the delegates from those countries.

The Economic Association has less varied interests, altho it also held a round-table conference on “Rural economics in relation to conservation.” At its first general session economic theory was treated both from the stand of “dynamic economics” and that of “theory of wages.” It was plain from the vigor of the discussion that economic theory, as doubtless it always will be, is in a state of flux, few men agreeing as to any one cure for the existing ills of the body politic. Another general session was held at the Chamber of Commerce, in connection with the financial magnates. Hereat was frankly considered “the causes and remedies for trusts,” in which the several divergent points of view, practical and theoretical, were squarely presented, presumably with mutual enlightenment.

For a young society, the Political Science people were exceptionally busy and vigorous. Ballot reform, the valuation of public service corporations (jointly with the economists), the relation of the State to labor (jointly with the labor legislation association), methods of instruction in municipal government and government of the Far East, were all duly considered, exhibiting a wide range of interest and possible future usefulness.

FRANK J. GOODNOW, Columbia University, first president American Political Science Association.

The sociologists were concerned (jointly with the Statistical Association) in such topics as the next census, the standardizing of units in studying public administration under democratic conditions, and the social marking system; and, individually, in the problems of methods in teaching psychological sociology, and in the religious factor in social revolution.

FRANKLIN H. GIDDINGS, Columbia University, vice-president Sociological Society.
FRANKLIN H. GIDDINGS, Columbia University, vice-president Sociological Society.

The labor legislation folk and the. statisticians chiefly held their sessions in conjunction with other bodies. For the most part the remainder of the participating societies confined themselves to listening to the usual presidential addresses and the transaction of necessary routine business.

HENRY W. FARNAM. Yale University, president of the American Association for Labor Legislation
HENRY W. FARNAM. Yale University, president of the American Association for Labor Legislation

The social side of the great conference was in every way notable. The entertainments offered to the thousand and more delegates ranged from receptions and breakfasts to formal dinners. The two most striking features of the daily and remarkably diverse program of hospitality were the great welcome meeting at Carnegie Hall, on Monday night, and the very attractive “historical playlet” and tableaux given by the ladies‘ reception committee at the Waldorf-Astoria (the headquarters of the several associations) on Wednesday night.

At Carnegie Hall the delegates were given the freedom of the city and State by Mayor McClellan and Governor Hughes. President Butler extended the welcome of Columbia University, and Mr. Joseph H. Choate and Professor Sloane spoke for the Committee of Arrangements. All were excellent addresses, but the Governor in particular rose to the occasion and earnestly commended the work of his hearers, who in their several ways are striving to find the correct principles underlying human society and seeking practically to apply these to the manifold problems of the State

In addition to the formal entertainments provided by the general committee were numerous unofficial attentions paid to various groups of visiting scholars. Among the most welcome of the unannounced gatherings of this character was the dinner given to fifty members of the American Antiquarian Society at the Metropolitan Club—the joyous forerunner, it was hoped, of annual banquets of these gentlemen at successive conferences of the American Historical Association. In all of these hospitalities practically every learned institution in the city, Columbia University properly leading, actively participated.

WALDO GIFFORD LELAND, A. M.,Carnegie Institution of Washington, secretary American Historical Association; THOMAS N. CARVER, Harvard University, secretary and treasurer, American Economic Association.
WALDO GIFFORD LELAND, A. M.,Carnegie Institution of Washington, secretary American Historical Association; THOMAS N. CARVER, Harvard University, secretary and treasurer, American Economic Association.

In every respect, professionally and socially, the great conference has been a marked success. The attendance was record-breaking, and the quality of the personnelle probably quite unexcelled in this country, even by the literary congresses at Chicago and St. Louis. The delegates, domestic and foreign, returned to their homes more than ever imprest [sic] by the hospitality, greatness and potentialities of America’s much favored metropolis.

Madison, Wis.

_______________________________ 

Source:  Reuben G. Thwaites “A Notable Gathering of Scholars,” The Independent, Vol. 68, January 6, 1910, pp. 7-14.

  • Copy in John Bates Clark Papers, Series II.4, Box 9, Folder 11, Report on American Economic Association’s 25th Anniversary, 1910”.
  • At www.archive.org the 68th volume of The Independent.    There I downloaded “Single Page Original JPS TAR” (warning: > 1 GB and then extracted the pages of the article, from which I have clipped the photos.
Categories
Economists Pennsylvania

Philadelphia. Summer Meeting of Economists. University Extension, 1894

We have here I think the first major extracurricular Summer Workshop in Economics for university graduates, post-docs and teachers of social studies and college instructors. Perhaps a dream-team of 1894 American economists (note the absence of Ely of Wisconsin, Taussig of Harvard and Laughlin of Chicago, though I don’t know if they might have been approached). The overview of Economic Science in America is really very interesting, both for ringing the exceptionalism bell and the light it casts on German graduate training in economics. The (approximate) ages of the lecturers in the Summer Meeting of Economists: Andrews (50), Clark (47), Giddings (39), Hadley (38), Jenks (38), Mayo-Smith (40), Patten(42), and Seligman (33).

Here the Announcement of the Summer Meeting of Economists by section:

Corps of Lecturers
Economic Science in America
To Graduates of Colleges
A Word to Students and Teachers of History
Statement of Courses
Program of Lectures
Preparatory Reading
More about University Extension

 

_________________________

Summer Meeting of Economists

IN CONNECTION WITH
The Second Session of the University Extension Summer Meeting,
JULY 2-28, 1894.
UNIVERSITY OF PENNSYLVANIA, PHILADELPHIA.

 

CORPS OF LECTURERS:

E. B. ANDREWS, Brown University; J. B. CLARK, Amherst College; F. H. GIDDINGS, Bryn Mawr College; A. T. HADLEY, Yale University; J. W. JENKS, Cornell University; R. MAYO-SMITH, Columbia College; S. N. PATTEN, University of Pennsylvania; E. R. A. SELIGMAN, Columbia College.

The Summer Meeting of Economists is held for the purpose of giving expression to present American Economic thought. The instructors are all identified with the recent remarkable expansion of Economic science and they have made important additions to its literature. The lectures which they will deliver in the Summer Meeting are intended primarily for students and teachers of economics, rather than for the diffusion of elementary knowledge.

The lectures will occupy about three hours daily for the four weeks. After each lecture an opportunity will be given for general discussion of the subject presented in the lecture. Besides the lectures and discussions, arrangement has been made for informal talks from several of the regular lecturers of the corps on methods of teaching. The program will be of interest to teachers of History, Political Science and similar subjects and to University students looking forward to any profession in which will be found useful a knowledge of economic science, and of the relations between economics and sociology on the one hand and economics and politics on the other.

Statement of the courses offered in the Economics Department of the Summer Meeting, program of lectures, and other information relating to the meeting, are contained in this number of the Bulletin. We present our readers also with a supplement with portraits of the lecturers in the Economics Department. An early number of the Bulletin, containing full announcement of other Summer Meeting Departments, will be sent on application.

Inaugural Lecture of Summer Meeting, Saturday evening, June 30, by Richard Watson Gilder, editor of the Century Magazine. Admission free by ticket.

Registration for Department of Economics, Ten Dollars.

Inclusive Ticket admitting to all Departments of Summer Meeting, Fifteen Dollars.

Instruction in other departments in Literature, Science, Architecture, Music, History, Mathematics, and Pedagogy.

For information concerning the Department of Economics or other Departments, address:
EDWARD T. DEVINE, Director, Fifteenth and Chestnut Streets, Philadelphia.

Back to top

_________________________

 

Economic Science in America.

The eight economists who constitute the corps of instructors in the Summer Meeting are representative of various phases of the new economics which, since the seventies, has swept like a wave over Europe and America. Until the appearance of General Walker’s “The Wages Question,” in 1876, there had been in the economic thought of the United States, two distinct and antagonistic schools. The orthodox English system had its chief interpretation in a translation of the Political Economy of J. B. Say, though there were American editions of the “Wealth of Nations” in its author’s lifetime, and the works of Ricardo, Malthus and McCulloch were familiar to students. After 1848, Mill’s Political Economy to some extent supplanted that of Say as the standard textbook. The native American economics dates not from Rae, who is properly of the English school, though he was a protectionist, and though by accident his book was published in Boston instead of in Scotland, nor from List, whose National System although contained in brief outline in a series of letters written in 1827 at Reading, Pa., had little or no influence on any American writers until it came through the medium of a French translation from the German work, but from Henry C. Carey, the Philadelphia economist, whose first book appeared in 1835.

The orthodox Political Economy, strongest in the New England colleges and in the South, stood for hard money and free trade. The Economics of Carey stood for protection and expansion of the currency. The former was in harmony with the naturally conservative temper of the English race, embodied, perhaps, more fully in Americans than in the English themselves, the latter was an expression of the spirit of enterprise which was called forth in the American people, or better, perhaps, forced upon them by their economic conditions. This first school of American thinkers was fortunate in thus being identified with what came to be known as the characteristic American spirit; it was unfortunate in its lack of conservatism on the question of money, and the resumption of specie payments in 1879, must be looked upon as a final victory for its opponents on the subject in which, if there is to be prosperity and progress, conservatism is essential.

Both these tendencies, that toward conservatism and that toward industrial enterprise, were characteristically American, but the one found its most natural expression in the English economics, the other in Carey’s system. Both schools influenced political thought. Daniel Webster in the Senate, would not have delivered his phillipic against “Political Economy” if that which he attacked had not had an active influence. Carey would not have found his German, French and Italian disciples if his system had been without scientific basis, and had been calculated like the essays of Mathew Carey, merely to exercise a temporary political influence. No doubt Carey cared much more about converting voters to his own views than he did about accomplishing a revolution in the science, yet he professed and, perhaps, came nearer than his critics have cared to admit in realizing both aims.

Such was the general condition of economic science in America when, in 1876, General Walker published his “Wages Question.” This book and the “Political Economy” of 1883, mark a new epoch. General Walker would doubtless prefer to be classed, if a classification is necessary, with the orthodox school of economists. He does not break with its earlier representatives on what they would have regarded as fundamental questions. His book naturally displaced Mill as the ordinary text at Oxford and Cambridge. Even in the discussion of distribution where Walker proposes his most radical departures, he starts with the Ricardian doctrine of rent, and declares, explicitly, that on this question he is a “Ricardian of Ricardians.” Nevertheless the appearance of these books in America mark the close of a long and, with the exceptions that have been noted, an almost barren epoch. Several textbooks, a few of them excellent for their purpose, had been prepared by American writers, but whatever originality they contained appeared chiefly in the omission, from the reproduction of the orthodox system, of particular dogmas which were felt to be inconsistent with the industrial conditions with which the writers were familiar.* Unlike his predecessors General Walker did not merely omit, he examined and analyzed those conditions, and when he was compelled to form new conclusions he neither attacked the old system entire, because of its errors, nor made the mistake of regarding his discoveries as slight modifications of detail. It has become clear that the changes were important though they were not revolutionary.

[footnote: *One exception to this statement must be made in favor of the clear and vigorous writings of Professor A. L. Perry, who did much to keep alive an interest in Political Economy in its languishing days and whose text-books have perhaps had more readers than those of any other American writer.]

In view of the introduction of a marked German influence almost immediately after Walker’s views became known, it is fair to regard the Political Economy of 1883 as the culmination of the influence of the “English economics” as it was also the most important contribution to economic science by the writers of that school since the appearance of the Political Economy of John Stuart Mill. If Walker belongs to the English school it must not be forgotten that his system is that of the English school remoulded by a man who understood and felt the full significance of American industrial conditions, and who was entirely free from any notion that Political Economy is a science comprising only a few ready-made principles and laws which are capable of statement in formal propositions.

Soon after the close of the Civil War, there was noticed a new interest in the scientific study of monetary and industrial, financial and economic problems. The pen of David A. Wells is to be credited in very large part with the creation of this new interest and with the diversion of public attention from the purely political to the economic aspects of the issues then in the public mind. His treatment of the probable issue of the war itself is typical of the character of his discussions. Far in advance of general public opinion, Mr. Wells discerned that the North would win because of its greater economic resources. This insistence on a controlling economic element in questions of public policy is always needed, but never more than in this period when political passions had dominated the country so completely and when a depreciated currency, a large national debt, and when a devastated South called for careful attention to sound policy in recuperative measures and in the new industrial activity which peace was to inaugurate. The reputation of the author of “Recent Economic Changes,” does not rest entirely upon the pamphlets which he issued at this time; but if we are to estimate rightly the causes of the intense interest in economics during the past twenty years we must not ignore their influence both on public opinion in general and particularly upon the young men who were interested in the great problems of the day, but were dissatisfied with the conventional political arguments.

And now began a new influence in American economics. The universities were unable to meet the demand for competent guidance in these studies and students began to seek such instruction abroad. The greater hospitality of the German universities, the unrivaled reputation of the founders of the German historical school of economics, and a feeling that more would be gained by foreign residence in a country whose institutions differ radically from our own were among the causes that combined to attract the American students almost exclusively to the German universities. Within a few years the American colleges began to give evidence of the new movement in the expansion of the curricula, the founding of new chairs, and the increase of students.

The English influence had been communicated by the importation and the republication of books. The German influence came through personal channels. This difference in the method of communication accounts in part for the astonishing differences in results. In the case of the English economics there were at hand standards of orthodoxy, a “system” in crystallized form. In the college classes there was produced a ready conviction of the correctness of certain principles and dogmas. In the case of the German influence such standards were lacking. Each new doctor of philosophy brought back the ideas of his instructors and associates in the foreign universities not in a formulated exact system, but in the form in which they had been impressed upon himself. He brought not so much a system of economics as an enthusiasm for independent research. The result is that no “system” has been transplanted by the newer economics, but only tendencies and a quickening impulse to activity in every branch of economic investigation, and already the impulse is seen to be of more importance than the particular tendencies.

When the American Economic Association was formed in 1885, as a tangible evidence of the new birth, a platform was adopted committing the association though not the individual members to favor increased industrial activity in the State, increased emphasis on the ethical element in economics, and increased attention to the historical method as distinguished from the deductive method which some of the leaders of the new organization believed to have been responsible for the decay of interest in economic science. But this platform was found to be too narrow, and in a few years it was discarded for a simple statement that any one might be chosen a member who is interested in the study of economics. General Walker was elected the first president of the association and continued in that office until 1892. Dr. Richard T. Ely, who served as secretary until the same year, labored indefatigablv in the interests of the association, building up its membership and also for a time editing its publications. In 1893 Professor Charles F. Dunbar, of Harvard, became president, and Professor Edward A. Ross, then of Cornell, secretary, and for the present year Professor John B. Clark, of Amherst, is president, and Professor J. W. Jenks, of CornelI, the secretary of the association. Professor F. H. Giddings succeeded Dr. Ely as chairman of the publication committee, a position which is held at present by Professor H. H. Powers, of Smith College.

The seven annual meetings of the American Economic Association have served as milestones of a rapid development of the science. Its position in the universities as a regular discipline of the university curriculum has become every year more secure. Thirty or forty professors and assistants are engaged in teaching its principles. Schools of finance and economy, departments of political and social science, lectureships on special economic topics abound. Every college has either an independent chair of Political Economy or a combined chair of economics and history, or some other subject. The larger universities have now organized, and in some instances liberally endowed these departments until they rival the best equipped corresponding departments of German, French and Italian universities. The movement which began in the seventies by sending dozens of students across the Atlantic, already bears fruit in courses of study sufficiently attractive to hold at home scores of students quite as ambitious and as discriminating.

There must be noticed finally, a new movement coming in part from the Austrian economists, in part from the English economist, Jevons, and in part originating with native-American writers, a movement which has been pronounced by some critics reactionary, but by its friends the most promising of all the various phases of our economic thought, the movement in the direction of deductive theory. Professor Patten’s “Premises of Political Economy” and Professor Clark’s “Philosophy of Wealth,” published respectively in 1885 and 1886, were its first fruits; and abundant evidences of its subsequent fruitfulness are to be found in the monographs of the Economic Association, in articles published in the economic journals and in the later literature generally. The translation of Böhm-Bawerk’s works by Professor William Smart, and the appearance of Professor Marshall’s “Principles of Economics,” both of which have had great influence in America, are landmarks in the progress of this movement. The “newer economics” has much to say of the relation between value and utility, the economic basis of prosperity and progress, the effects of dynamic forces. It seeks a new correlation of the social sciences, and in its scheme of human progress does not omit to take account of costs, and to distinguish sharply individual costs or “expenses” from social costs, which latter item it measures subjectively and ventures to compare directly with utilities or “satisfactions” as a means of determining the, social surplus.

One group of writers belonging with the newer movement, but devoting its energies directly to sociological studies, gives promise of rescuing that much misconceived branch of study from the hands of its injudicious representatives and putting it upon a high scientific plane. Professor F. H. Giddings who will become Professor of Sociology in Columbia College on July 1 of the present year, is the foremost scholar of this group, and the first man in any American university to occupy a chair with this designation. The future of economic science in American universities is bright with promise of scholarly and useful work. The attitude of the university world and of the public toward what is after all a new science, is all that could be desired. One indication of the present healthy and vigorous condition of this branch of science in American universities, is the quality and quantity of its scientific literature. The “younger economists” are already mature in years and in scholarship, and the publications of the American Economic Association, of the American Academy of Political and Social Science, of the separate universities in their series of Political Economy, Public Law, of studies in Historical and Political Science, etc., add to the stock of valuable economic literature no less than the regular issues of such quarterly journals as the Yale Review, the Journal of Political Economy the Political Science Quarterly and the Quarterly Journal of Economics, or the bi-monthly journal, the Annals of the American Academy of Political and Social Science.

The Summer Meeting of economists, of which announcement is made in full in this number of the Bulletin, may well become a great landmark, an emphatic sign of the golden opportunities awaiting students who turn their attention seriously to these problems.

Edward T. Devine.
The American Society for the Extension of University Teaching

 

Back to top

 

_________________________

To Graduates of Colleges.

The increasing tendency toward specialization in the upper college classes makes it difficult for the college student to secure an acquaintance with as many different subjects properly falling within the college curriculum as every cultured man or woman considers desirable. College students who have specialized on economics and finance, may have left serious gaps in their knowledge of the physical sciences and vice versa, while both may have neglected the humanities, belles lettres and philosophy. University Extension courses in the local centres have already been eagerly utilized by many college graduates to supply such deficiencies and even if the purpose of the movement be chiefly, as some contend, to carry university privileges to those that have them not, it is attaining that purpose in meeting just such demands. The University Extension Summer Meeting offers similar opportunities. It takes place in a vacation month. It calls to the lecture room eminent specialists in many departments of university study. The student who is proficient in literature may hear brief courses in science or philosophy. The teacher who is thoroughly familiar with his special subject may make a careful study of a pedagogical system, or may refresh his intellectual powers by attacking vigorously a new line of study. It is true that every teacher should at some time or other have “specialized” to such an extent as to understand and to share somewhat the modern university spirit, but it is also true that modern culture demands of persons trained in a special subject a sufficient knowledge of other and entirely distinct fields of knowledge to awaken an intelligent interesting the achievements of the specialists of those fields.

In two ways therefore the Summer Meeting may be of use to college graduates. It will give to the student of a particular subject a favorable opportunity to supplement his specialized knowledge by a general—not necessarily a superficial—knowledge of other subjects. It will enable the student who wishes to broaden his knowledge of his own subject to do so by acquainting himself at first hand with a knowledge of the systems held and the methods employed by teachers of that subject in other institutions. It will be of great advantage for instance for the young man who has studied Political Economy in the University of Pennsylvania, or Johns Hopkins, or Cornell, to hear lecturers from Yale and Columbia discuss the same subject; and to become acquainted with the men who have studied that subject in those institutions, and vice versa. No student of history in an Eastern institution could fail to profit by the course of lectures on the Place of the West in our history by the professor of History in the University of Wisconsin. Graduates of normal schools, or of departments of pedagogy will derive more benefit than any others from the course on the Herbartian pedagogy by one who vigorously champions the system and has studied it at its fountain head in the University of Jena, and from the lectures on child study and its pedagogical value by the specialist who has been prosecuting an investigation of that subject in the State Normal School of Massachusetts, and under the direction of Dr. G. Stanley Hall, of Clark University.

This is the great advantage of the Summer Meeting over a summer session of corresponding length in any single university. We plan not a summer school, but a meeting, a mingling of students and lecturers, a gathering with all the definiteness of aim and of program which characterizes a school or the summer term of a university, but with the added advantages of a University Extension spirit as an esprit de corps and a union of progressive elements from many universities in an elective system of lectures and classes.

Back to top

 

_________________________

A Word to Students and Teachers of History.

The now famous report of the Committee of Ten on Secondary School Studies contains the following resolutions from the “Conference on History, Civil Government and Political Economy:”

Resolved: That formal instruction in Political Economy be omitted from the school program; but that economic subjects be treated in connection with other pertinent subjects. (Resolution 9.)

Resolved: That no formal instruction in Political Economy be given in the secondary schools, but that in connection particularly with United States History, Civil Government and Commercial Geography, instruction be given in those economic topics, a knowledge of which is essential to the understanding of our economic life and development. (Resolution 30.)

Accompanying the resolutions is a memorandum in which it is stated that “in making these recommendations the Conference does not intend to suggest that less time than is customary be given to Political Economy or that less emphasis be given to its importance as a study in the high schools;” and the report of the Conference elsewhere contains the following significant statements: “The methods of teaching the economic principles thus indicated must be left to the discretion of the teacher. It is a subject in which textbook work is particularly inefficient, and no teacher ought to undertake the work who has not had some training in economic reasoning.”

The unavoidable inference from these resolutions and recommendations is that every teacher of history, civil government, or commercial geography in the schools of secondary grade should have some opportunity for training in economic reasoning. Since, in the opinion of the committee, there are no “proper text-books for high school use” it becomes of importance that teachers should become familiar at first hand with the vital principles as taught by the best economic authorities. A few years ago it was thought necessary to visit the German or other foreign universities for such contact with leaders of economic thought. At present the men who are teaching these subjects in Harvard, Yale, Columbia, Johns Hopkins, Cornell and Pennsylvania, are scholars of international reputation and are original contributors to economic science.

The program of the Department of Economics in the Summer Meeting is framed with the express end of giving a rapid view of such principles as are by the economists deemed essential, and illustrating the methods of instruction in vogue in the leading universities. Those who expect to teach Political Economy in university, college or secondary school, those who are expecting to give instruction in history, civil government or commercial geography, and those who are regularly engaged in teaching these branches are cordially invited to examine carefully the courses announced for the Summer Meeting of Economists, and to avail themselves of the opportunities offered by the meeting.

The president, first vice-president and secretary of the American Economic Association are included in the corps of instructors. Among higher institutions Amherst, Brown, Bryn Mawr, Columbia, Cornell, Johns Hopkins, Pennsylvania and Yale are represented. One of the instructors is a university president, the others are university or college professors. All have written important books or monographs on economic subjects. All have national and even international scientific reputation. All are associated with the recent notable development of economic science, and the corresponding expansion of economic departments in the higher educational institutions. They do not however, all represent the same or similar tendencies. The corps includes the two or three economists who have done most among American writers to emphasize the importance of deductive work, and the necessity of reforming economic theory, but it also contains the two or three men who would be first thought of in connection with such practical topics as public finance, railways and trusts.

It is difficult to imagine a more profitable method of spending a vacation month for a person who has a professional interest in acquainting himself with the methods used and the conclusions held by the men whose scientific reputation and academic standing entitle them to speak with a certain degree of authority. If the Committee of Ten and the Conference on History, Civil Government and Political Economy are correct in their view, this includes not merely the teacher of Political Economy and Political Science, but also teaches of such allied subjects as commercial geography, civil government and history.

The above considerations are strengthened by the fact that parallel with these economic course there will be instruction in European and American history by such distinguished and competent lecturers as Professor John Bach McMaster and Mr. W. H. Munro, of the University of Pennsylvania, Professor Frederick J. Turner, of the University of Wisconsin, Professor W. H. Mace, of Syracuse University and Dr. Edward Everett Hale, of Boston. A fuller announcement of these courses will be sent on application. Round-Table Conferences on the teaching of history in secondary schools will be conducted by Professor Ray Greene Huling, of Boston, and Professor Edward G. Bourne, of Adelbert College, both of whom were members of the conference from whose report extracts have been made.

Back to top

 

_________________________

Statement of Courses in the
Summer Meeting of Economists,

July 2-28, 1894.

 

Course I—Money. By E. Benjamin Andrews, LL. D., President of Brown University.

Five Lectures—July 16-20. (1) Money and the Times; (2) England’s Monetary Experiment in India; (3) “Counter” and Quality in Monetary Theory; (4) What Fixes Prices; (5) Labor as a Standard of Value.

Course II—Distribution. By J. B. Clark, Ph. D., Professor of Political Economy in Amherst College, and Lecturer in Johns Hopkins University.

Ten Lectures—July 2-18. (1) Normal Distribution equivalent to Proportionate Production; (2) The Relation of the Law of Value to the Law of Wages and Interest; (3) The Social Law of Value; (4) Groups and Sub-groups in Industrial Society; (5) The Nature of Capital and the Source of Wages and Interest; (6) The Static Law of Distribution ; (7) Dynamic Forces and their Effects; (8) The Origin and the Distribution of Normal Profits; (9) Trusts and Public Policy; (10) Labor Unions and Public Policy.

Course III—Scientific Subdivision of Political Economy. By F. H. Giddings, M. A., Professor of Political Science in Bryn Mawr College and Professor elect of Sociology in Columbia College.

Five Lecture»—July 2-7. (1) The Conception and Definition of Political Economy; (2) The Concepts of Utility, Cost and Value; (3) The Theory of Consumption; (4) The Theory of Production; (5) The Theory of Relative Values.

Course IV—Theories of Population. By Arthur T. Hadley, M. A., Professor of Political Economy in Yale University.

Two Lectures—July 5, 6.

Course V—Relations of Economics and Politics. By J. W. Jenks, Ph. D, Professor of Political Economy and Civil and Social Institutions in Cornell University.

Five Lectures—July 16-20. (1) The Nature and Scope of Economics and of Politics Compared; (2) Influence of Economic Conditions upon Political Constitutions; (3) The Influence of Economic Conditions and Theories upon Certain Social and Legal Institutions not Primarily Political; (4) The Influence of Present Economic Conditions and Beliefs upon Present Political Methods and Doctrine; (5) The Political Reforms that would be of most Economic Advantage.

Course VI—Ethnical Basis for Social Progress in the United States. By Richmond Mayo-Smith, Ph. D., Professor of Political Economy and Social Science in Columbia College.

Three Lectures— July 24-26. (1) Theories of Mixture of Races and Nationalities and Application to the United States; (2) Assimilating Influence of Climate and Intermarriages; (3) Assimilating Influence of Social Environment.

Course VII—Introduction to the Ricardian Economics. By Simon N. Patten, Ph. D., Professor of Political Economy in the University of Pennsylvania.

Five Lectures—July 9-13.

Course VIII—Premises of Political Economy. By Simon N. Patten, Ph. D.

Five Lectures—July 16-20.

Course IX—Theory of Dynamic Economics. By Simon X. Patten, Ph. D.

Five Lectures—July 23-27.

Course X—Public Finance. By Edwin R. A. Seligman, Ph. D., Professor of Political Economy and Finance in Columbia College.

Five Lectures—July 23-27. (1) The Development of Taxation; (2) The Effects of Taxation; (3) The Basis of Taxation; (4) The Principles of Taxation; (5) The Single Tax.

Course XI—Various Phases of the Money Question. By Professor Clark, Professor Giddings, Professor Patten and Professor Seligman.

Address “The Monetary Conference of 1 892.” By President Andrews. July 19.

Address on Methods of Teaching Political Economy. By members of the corps of lecturers.

Discussion of the subjects presented in each of the various courses by those in attendance. The lecture will usually last for sixty minutes, and the discussion for thirty minutes. An hour and a half is allowed for each exercise.

Back to top

 

_________________________

 

Program of Lectures.
Summer Meeting of Economists.

[For program of other departments apply to the Director.]

July 2.

8.30 A. M.—Professor Giddings.

The Conception and Definition of Political Economy.

10 A. M.—Professor Clark.

Normal Distribution Equivalent to Proportionate Production.

July 3.

8.30 A. M.—Professor Giddings.

The Concepts of Utilitv, Cost and Value.

10 A. M— Professor Clark.

The Relation of the Law of Value to the Law of Wages and Interest.

July 4.

10 A. M.—Address by Edward Everett Hale, D. D., in the University Library.

July 5.

8.30 A. M.—Professor Giddings.

The Theory of Consumption.

10 A. M.—Professor Clark.

The Social Law of Value.

11.30 A. M.—Professor Hadley.

Theories of Population.

July 6.

8.30 A. M.—Professor Giddings.

The Theory of Production.

10 A. M.—Professor Clark.

Groups and Sub-Groups in Industrial Society.

11.30 A. M.—Professor Hadley.

Theories of Population.

5 P. M.—Professor Clark.

An Ideal Standard of Value.

July 7.

8.30 A. M.—Professor Giddings.

The Theory of Relative Values.

10 A. M—Professor Clark.

The Nature of Capital and the Sources of Wages and Interest.

July 9.

8.30 A. M.—Professor Patten.

Ricardian System of Economics.

10 A. M.—Professor Clark.

The Static Law of Distribution.

5 P. M.—Address on Methods.

July 10

8.30 A. M.—Professor Patten.

Ricardo’s Theory of Distribution.

10 A. M.—Professor Clark.

Dynamic Forces and their Effects.

5 P. M.—Address on Methods.

July 11.

8.30 A. M.—Professor Patten.

Ricardo’s Theory of Money.

10 A. M— Professor Clark.

The Origin and Distribution of Normal Profits.

5 P. M.—Address on Methods.

July 12.

8.30 A. M— Professor Patten.

The Confusion of Industrial and Monetary Problems.

10 A. M.—Professor Clark.

Trusts and Public Policy.

July 13.

8.30 A. M.—Professor Patten.

Ricardian System of Economics.

10 A. M.—Professor Clark.

Labor Unions and Public Policy.

July 16.

8.30 A. M.—Professor Patten.

Premises of Political Economy.

10 A. M.—President Andrews.

Money and the Times.

July 17.

8.30 A. M.—Professor Patten.

Premises of Political Economy.

10 A. M.—President Andrews.

England’s Monetary Experiment in India.

July 18.

8.30 A. M.—Professor Patten.

The Stability of Prices.

10 A. M.—President Andrews.

“Counter” and Quality in Monetary Theory.

July 19.

8.30 A. M.—Professor Patten.

The Law of Diminishing Returns.

10 A. M.—President Andrews.

What Fixes Prices?

8 P. M.—President Andrews.

Monetary Conference.

July 20.

8.30 A. M.—Professor Patten.

The Consumption of Wealth.

10 A. M.—President Andrews.

Labor as a Standard of Value.

July 23.

8.30 A. M.—Professor Patten.

Theory of Dynamic Economics.

5 P. M.—Professor Seligman.

Development of Taxation.

8 P. M.—Professor Jenks.

Nature and Scope of Economics and Politics Compared.

July 24.

8.30 A. M.—Professor Patten.

Theory of Dynamic Economics.

10 A. M.—Professor Seligman.

The Effects of Taxation.

11.30 A. M.—Professor Mayo-Smith.

Theories of Mixture of Races, and Nationalities.

8 P. M.—Professor Jenks. Influence of Economic Conditions upon Political Constitutions.

July 25.

8.30 A. M — Professor Patten.

Theory of Dynamic Economics.

10 A. M.—Professor Seligman.

Basis of Taxation.

11.30 A. M.—Professor Mayo-Smith.

Assimilating Influences of Climate and Intermarriages.

8 P. M.—Professor Jenks.

Influence of Economic Conditions and Theories upon Certain Social and Legal Institutions not Primarily Political.

July 26.

8.30 A. M.—Professor Patten.

Theory of Dynamic Economics.

10 A. M.—Professor Seligman.

The Principles of Taxation.

11.30 A. M.— Professor Mayo-Smith.

Assimilating Influences of Social Environment.

8 P. M.—Professor Jenks.

Influence of Present Economic Conditions and Beliefs upon Present Political Methods and Doctrine.

July 27.

8.30 A. M — Professor Patten.

Theory of Dynamic Economics.

10 A. M.—Professor Seligman.

The Single Tax.

11.30 A. M.—Professor Jenks.

The Political Reforms that would be of Most Economic Advantage.

 

Back to top

_________________________

 

Preparatory Reading.

Those who expect to attend the sessions of the Summer Meeting of Economists will find it of advantage to possess a knowledge of the elements of the science such as may be obtained by the study of Walker’s Political Economy, Marshall’s Principles of Economics or Mill’s Political Economy.

In special preparation for the meeting, Giddings’ The Theory of Sociology (in press) will be found useful. In special preparation for Course I, students may read Andrews’ An Honest Dollar, and Nicholson’s Money and Monetary Problems; for Courses II and III, Clark’s Philosophy of Wealth, and Modern Distributive Process, by Clark and Giddings; for Course VII, Patten’s The Interpretation of Ricardo in Quarterly Journal of Economics for April, 1893; for Course VIII, Patten’s Premises of Political Economy; for Course IX, The Theory of Dynamic Economics.

Back to top

_________________________

Source: American Society for the Extension of University Teaching. The University Extension Bulletin. Vol. I, No. 8. Philadelphia: May 10, 1894.

Image Source: American Society for the Extension of University Teaching. Supplement to the The University Extension Bulletin. Vol. I, No. 8. Philadelphia: May 10, 1894. Copy found in Box 2 of Franklin Henry Giddings Papers, Columbia Rare Book & Manuscript Library, Folder “Photographs”.

 

More on what University Extension was all about.

 

Back to top